Med Surg MIDTERM - Testbankgo Questions

Pataasin ang iyong marka sa homework at exams ngayon gamit ang Quizwiz!

The nurse is teaching a client about the development of leukemia. What statement should be included in the teaching plan?

"Chronic leukemia develops slowly."

The nurse is teaching the client about consolidation. What statement should be included in the teaching plan?

"Consolidation therapy is administered to reduce the chance of leukemia recurrence."

A nurse is planning the care of a patient with herpes zoster. What medication, if administered within the first 24 hours of the initial eruption, can arrest herpes zoster?

Acyclovir (Zovirax)

Which blood test confirms the presence of antibodies to HIV?

ELISA

A young student is brought to the school nurse after falling off a swing. The nurse is documenting that the child has bruising on the lateral aspect of the right arm. What term will the nurse use to describe bruising on the skin in documentation?

Ecchymoses

A clinic nurse is caring for a patient newly diagnosed with fibromyalgia. When developing a care plan for this patient, what would be a priority nursing diagnosis for this patient?

Fatigue Related to Pain

Infants with DiGeorge syndrome have which type of endocrine disorder?

Hypoparathyroidism

The nurse is admitting an oncology patient to the unit prior to surgery. The nurse reads in the electronic health record that the patient has just finished radiation therapy. With knowledge of the consequent health risks, the nurse should prioritize assessments related to what health problem?

Impaired wound healing

Which medication classification is known to inhibit prostaglandin synthesis or release?

NSAID (in large doses)

In which process is the antigen-antibody molecule coated with a sticky substance that facilitates phagocytosis?

Opsonization

When do most perinatal HIV infections occur?

after exposure during delivery

SCID

autosomal dominant

Chronic illnesses may contribute to immune system impairment in various ways. Renal failure is associated with

deficiency in circulating lymphocytes

During which stage of the immune response does the circulating lymphocytes containing the antigenic message return to the nearest lymph node?

proliferation

The lower the patients viral load..

the longer the survival time

You are the nurse caring for a 91-year-old patient admitted to the hospital for a fall. The patient complains of urge incontinence and tells you he most often falls when he tries to get to the bathroom in his home. You identify the nursing diagnosis of risk for falls related to impaired mobility and urinary incontinence. The older adults risk for falls is considered to be which of the following?

A geriatric syndrome

A nurse is caring for an 86-year-old female patient who has become increasingly frail and unsteady on her feet. During the assessment, the patient indicates that she has fallen three times in the month, though she has not yet suffered an injury. The nurse should take action in the knowledge that this patient is at a high risk for what health problem?

A hip fracture

A nurse on an oncology unit has arranged for an individual to lead meditation exercises for patients who are interested in this nonpharmacological method of pain control. The nurse should recognize the use of what category of nonpharmacological intervention?

A mind-body method

The nurse is planning the care of a patient with a nutritional deficit and a diagnosis of megaloblastic anemia. The nurse should recognize that this patients health problem is due to what?

Abnormalities in the structure and function RBCs

A patient requires a full-thickness graft to cover a chronic wound. How is the donor site selected?

An area matching the color and texture of the skin at the surgical site is selected.

Which assessment should be completed if an immune dysfunction in the neurosensory system is suspected?

Assess for ataxia using the finger-to-nose test and heel-to-shin test

The medical nurse is aware that patients with sickle cell anemia benefit from understanding what situations can precipitate a sickle cell crisis. When teaching a patient with sickle cell anemia about strategies to prevent crises, what measures should the nurse recommend?

Avoiding cold temperatures and ensuring sufficient hydration

An interdisciplinary team has been commissioned to create policies and procedures aimed at preventing acute hemolytic transfusion reactions. What action has the greatest potential to reduce the risk of this transfusion reaction?

Be vigilant in identifying the patient and the blood component.

A 52-year-old female patient is receiving care on the oncology unit for breast cancer that has metastasized to her lungs and liver. When addressing the patients pain in her plan of nursing care, the nurse should consider what characteristic of cancer pain?

Cancer pain can be acute or chronic and it typically requires comparatively high doses of pain medications.

The nurse in an ambulatory care center is admitting an older adult patient who has bright red moles on the skin. Benign changes in elderly skin that appear as bright red moles are termed what?

Cherry angiomas

A gerontologic nurse is teaching students about the high incidence and prevalence of dehydration in older adults. What factors contribute to this phenomenon? Select all that apply.

Decreased kidney mass Decreased renal blood flow Decreased excretion of potassium

An 84-year-old patient has returned from the post-anesthetic care unit (PACU) following hip arthroplasty. The patient is oriented to name only. The patients family is very upset because, before having surgery, the patient had no cognitive deficits. The patient is subsequently diagnosed with postoperative delirium. What should the nurse explain to the patients family?

Delirium of this type is treatable and her cognition will return to previous levels.

A nurse knows that patients with invisible disabilities like chronic pain often feel that their chronic conditions are more challenging to deal with than more visible disabilities. Why would they feel this way?

Despite appearances, invisible disabilities can be as disabling as visible disabilities.

The nurse is providing care for a patient with chronic obstructive pulmonary disease. When describing the process of respiration the nurse explains how oxygen and carbon dioxide are exchanged between the pulmonary capillaries and the alveoli. The nurse is describing what process?

Diffusion

The nurse caring for a patient post colon resection is assessing the patient on the second postoperative day. The nasogastric tube (NG) remains patent and continues at low intermittent wall suction. The IV is patent and infusing at 125 mL/hr. The patient reports pain at the incision site rated at a 3 on a 0-to-10 rating scale. During your initial shift assessment, the patient complains of cramps in her legs and a tingling sensation in her feet. Your assessment indicates decreased deep tendon reflexes (DTRs) and you suspect the patient has hypokalemia. What other sign or symptom would you expect this patient to exhibit?

Dilute urine

You are caring for a patient admitted with a diagnosis of acute kidney injury. When you review your patients most recent laboratory reports, you note that the patients magnesium levels are high. You should prioritize assessment for which of the following health problems?

Diminished deep tendon reflexes

A nurse is reviewing gerontologic considerations relating to the care of patients with dermatologic problems. What vulnerability results from the age-related loss of subcutaneous tissue?

Diminished protection of tissues and organs

A nurse is preparing a patient for allergy skin testing. Which of the following precautionary steps is most important for the nurse to follow?

Emergency equipment should be readily available.

A hospice nurse is well aware of how difficult it is to deal with others pain on a daily basis. This nurse should put healthy practices into place to guard against what outcome?

Emotional exhaustion

A nurse is providing care for a patient who has developed Kaposis sarcoma secondary to HIV infection. The nurse should be aware that this form of malignancy originates in what part of the body?

Endothelial cells lining small blood vessels

A nurse is caring for a patient who is suspected of having giant cell arteritis (GCA). What laboratory tests are most useful in diagnosing this rheumatic disorder? Select all that apply.

Erythrocyte sedimentation rate C-reactive protein

When assessing a female client with a disorder of the hematopoietic or the lymphatic system, which assessment is most essential?

Health history, such as bleeding, fatigue, or fainting

In your role as a school nurse, you are presenting at a high school health fair and are promoting the benefits of maintaining a healthy body weight. You should refer to reductions in the risks of what diseases? Select all that apply.

Heart disease Stroke Diabetes Hypertension

A gerontologic nurse practitioner provides primary care for a large number of older adults who are living with various forms of cardiovascular disease. This nurse is well aware that heart disease is the leading cause of death in the aged. What is an age-related physiological change that contributes to this trend?

Heart muscle and arteries lose their elasticity.

A nurse at a blood donation clinic has completed the collection of blood from a woman. The woman states that she feels lightheaded and she appears visibly pale. What is the nurses most appropriate action?

Help her into a sitting position with her head lowered below her knees.

A 37-year-old woman with multiple sclerosis is married and has three children. The nurse has worked extensively with the woman and her family to plan appropriate care. What is the nurses most important role with this patient?

Help the patient develop strategies to implement treatment regimens

A patients electronic health record states that the patient receives regular transfusions of factor IX. The nurse would be justified in suspecting that this patient has what diagnosis?

Hemophilia

A patient, 25 years of age, comes to the emergency department complaining of excessive bleeding from a cut sustained when cleaning a knife. Blood work shows a prolonged PT but a vitamin K deficiency is ruled out. When assessing the patient, areas of ecchymosis are noted on other areas of the body. Which of the following is the most plausible cause of the patients signs and symptoms?

Hepatic dysfunction

You are working on a burns unit and one of your acutely ill patients is exhibiting signs and symptoms of third spacing. Based on this change in status, you should expect the patient to exhibit signs and symptoms of what imbalance?

Hypovolemia

A patient with multiple food and environmental allergies tells the nurse that he is frustrated and angry about having to be so watchful all the time and wonders if it is really worth it. What would be the nurses best response?

I can only imagine how you feel. Would you like to talk about it?

An elderly female patient who is bedridden is admitted to the unit because of a pressure ulcer that can no longer be treated in a community setting. During your assessment of the patient, you find that the ulcer extends into the muscle and bone. At what stage would document this ulcer?

IV

A patient has recently been diagnosed with type 2 diabetes. The patient is clinically obese and has a sedentary lifestyle. How can the nurse best begin to help the patient increase his activity level?

Identify barriers with the patient that inhibit his lifestyle change.

A patients daughter has asked the nurse about helping him end his terrible suffering. The nurse is aware of the ANA Position Statement on Assisted Suicide, which clearly states that nursing participation in assisted suicide is a violation of the Code for Nurses. What does the Position Statement further stress?

Identifying patient and family concerns and fears

When writing a plan of care for a patient with psoriasis, the nurse would know that an appropriate nursing diagnosis for this patient would be what?

Impaired Skin Integrity Related to Scaly Lesions

The nurse is creating a care plan for a patient suffering from allergic rhinitis. Which of the following outcomes should the nurse identify?

Improved coping with lifestyle modifications

An international nurse has noted that a trend in developing countries is a decrease in mortality from some acute conditions. This has corresponded with an increase in the incidence and prevalence of chronic diseases. What has contributed to this decrease in mortality from some acute conditions?

Improved nutrition

The nurse educating a patient with anemia is describing the process of RBC production. When the patients kidneys sense a low level of oxygen in circulating blood, what physiologic response is initiated?

Increased production of erythropoietin

You are the nurse evaluating a newly admitted patients laboratory results, which include several values that are outside of reference ranges. Which of the following would cause the release of antidiuretic hormone (ADH)?

Increased serum sodium

A patient has a documented history of allergies presents to the clinic. She states that she is frustrated by her chronic nasal congestion, anosmia (inability to smell) and inability to concentrate. The nurse should identify which of the following nursing diagnoses?

Ineffective Individual Coping with Chronicity of Condition and Need for Environmental Modification

The nurse on the pediatric unit is caring for a 10-year-old boy with a diagnosis of hemophilia. The nurse knows that a priority nursing diagnosis for a patient with hemophilia is what?

Ineffective coping

A nurse is planning the care of a patient with a diagnosis of sickle cell disease who has been admitted for the treatment of an acute vaso-occlusive crisis. What nursing diagnosis should the nurse prioritize in the patients plan of care?

Ineffective tissue perfusion related to thrombosis

You are performing an admission assessment on an older adult patient newly admitted for end-stage liver disease. What principle should guide your assessment of the patients skin turgor?

Inelastic skin turgor is a normal part of aging.

A patient has a diagnosis of rheumatoid arthritis and the primary care provider has now prescribed cyclophosphamide (Cytoxan). The nurses subsequent assessments should address what potential adverse effect?

Infection

You are making initial shift assessments on your patients. While assessing one patients peripheral IV site, you note edema around the insertion site. How should you document this complication related to IV therapy?

Infiltration

Which treatment therapy could be used to treat chronic hepatitis C?

Interferon

The interface model of disability is being used to plan the care of a patient who is living with the effects of a stroke. Why should the nurse prioritize this model?

It promotes the idea that patients are capable and responsible.

The nurse is providing patient teaching to a patient with early stage Alzheimers disease (AD) and her family. The patient has been prescribed donepezil hydrochloride (Aricept). What should the nurse explain to the patient and family about this drug?

It slows the progression of AD.

A nurse is performing the health history and physical assessment of a patient who has a diagnosis of rheumatoid arthritis (RA). What assessment finding is most consistent with the clinical presentation of RA?

Joint stiffness, especially in the morning

Which substance may be used to lubricate a condom?

K-Y jelly

A patient has developed severe contact dermatitis with burning, itching, cracking, and peeling of the skin on her hands. What should the nurse teach the patient to do?

Keep her hands well-moisturized at all times.

A 60-year-old patient with a diagnosis of prostate cancer is scheduled to have an interstitial implant for high-dose radiation (HDR). What safety measure should the nurse include in this patients subsequent plan of care?

Limit the time that visitors spend at the patients bedside.

The nurse who is a member of the palliative care team is assessing a patient. The patient indicates that he has been saving his PRN analgesics until the pain is intense because his pain control has been inadequate. What teaching should the nurse do with this patient?

Medication should be taken when pain levels are low so the pain is easier to reduce.

The nurse is assessing a new patient with complaints of overwhelming fatigue and a sore tongue that is visibly smooth and beefy red. This patient is demonstrating signs and symptoms associated with what form of what hematologic disorder?

Megaloblastic anemia

A patient is undergoing diagnostic testing to determine the etiology of recent joint pain. The patient asks the nurse about the difference between osteoarthritis (OA) and rheumatoid arthritis (RA). What is the best response by the nurse?

OA is a considered a noninflammatory joint disease. RA is characterized by inflamed, swollen joints.

What nursing action is essential before EIA testing is performed?

Obtaining a general consent for medical care from the patient

An 83-year-old woman was diagnosed with Alzheimers disease 2 years ago and the disease has progressed at an increasing pace in recent months. The patient has lost 16 pounds over the past 3 months, leading to a nursing diagnosis of Imbalanced Nutrition: Less than Body Requirements. What intervention should the nurse include in this patients plan of care?

Offer the patient only one food item at a time to promote focused eating

The nurse is providing care for an older adult who has a hematologic disorder. What age-related change in hematologic function should the nurse integrate into care planning?

Older adults are less able to increase blood cell production when demand suddenly increases.

The nurse is caring for a patient with metastatic bone cancer. The patient asks the nurse why he has had to keep getting larger doses of his pain medication, although they do not seem to affect him. What is the nurses best response?

Over time you become more tolerant of the drug.

A patient has been admitted to a medical unit with a diagnosis of polymyalgia rheumatica (PMR). The nurse should be aware of what aspects of PMR? Select all that apply.

PMR has an association with the genetic marker HLA-DR4. Immunoglobulin deposits occur in PMR. PMR occurs predominately in Caucasians.

A patient with a history of cirrhosis is admitted to the ICU with a diagnosis of bleeding esophageal varices; an attempt to stop the bleeding has been only partially successful. What would the critical care nurse expect the care team to order for this patient?

Packed red blood cells (PRBCs)

A nurse is caring for a patient with severe anemia. The patient is tachycardic and complains of dizziness and exertional dyspnea. The nurse knows that in an effort to deliver more blood to hypoxic tissue, the workload on the heart is increased. What signs and symptoms might develop if this patient goes into heart failure?

Peripheral edema

A majority f patients with CVID develop which type of anemia?

Pernicious

A patients blood work reveals a platelet level of 17,000/mm3. When inspecting the patients integumentary system, what finding would be most consistent with this platelet level?

Petechiae

A patient with a chronic diabetic wound is being discharged after receiving a skin graft to aid wound healing. What direction should the nurse include in home care instructions?

Protect the graft from direct sunlight and temperature extremes.

A patient is brought to the emergency department (ED) in a state of anaphylaxis. What is the ED nurses priority for care?

Protect the patients airway.

A patients blistering disorder has resulted in the formation of multiple lesions in the patients mouth. What intervention should be included in the patients plan of care?

Provide chlorhexidine solution for rinsing the patients mouth.

You are the home health nurse caring for a homebound client who is terminally ill. You are delivering a patient-controlled analgesia (PCA) pump to the patient at your visit today. The family members will be taking care of the patient. What would your priority nursing interventions be for this visit?

Provide patient and family teaching regarding the operation of the pump, monitoring the IV site, and knowing the side effects of the medication.

A nurse is creating a plan of care for an oncology patient and one of the identified nursing diagnoses is risk for infection related to myelosuppression. What intervention addresses the leading cause of infection-related death in oncology patients?

Provide skin care to maintain skin integrity.

A 67-year-old woman experienced the death of her husband from a sudden myocardial infarction 5 weeks ago. The nurse recognizes that the woman will be going through the process of mourning for an extended period of time. What processes of mourning will allow the woman to accommodate the loss in a healthy way? Select all that apply.

Reinvesting in new relationships at the appropriate time Reminiscing about the relationship she had with her husband Relinquishing old attachments to her husband at the appropriate time

Based on a patients vague explanations for recurring injuries, the nurse suspects that a community-dwelling older adult may be the victim of abuse. What is the nurses primary responsibility?

Report the findings to adult protective services.

A patients diagnosis of atrial fibrillation has prompted the primary care provider to prescribe warfarin (Coumadin), an anticoagulant. When assessing the therapeutic response to this medication, what is the nurses most appropriate action?

Review the patients international normalized ratio (INR).

A patients decreased mobility is ultimately the result of an autoimmune reaction originating in the synovial tissue, which caused the formation of pannus. This patient has been diagnosed with what health problem?

Rheumatoid arthritis (RA)

After a sudden decline in cognition, a 77-year-old man who has been diagnosed with vascular dementia is receiving care in his home. To reduce this mans risk of future infarcts, what action should the nurse most strongly encourage?

Rigorous control of the patients blood pressure and serum lipid levels

The nurse is planning the care of a patient who has a diagnosis of atopic dermatitis, which commonly affects both of her hands and forearms. What risk nursing diagnosis should the nurse include in the patients care plan?

Risk for Disturbed Body Image Related to Skin Lesions

The nurse is providing care for a patient who has a diagnosis of hereditary angioedema. When planning this patients care, what nursing diagnosis should be prioritized?

Risk for Impaired Gas Exchange Related to Airway Obstruction

A nurse is caring for a patient who undergoing preliminary testing for a hematologic disorder. What sign or symptom most likely suggests a potential hematologic disorder?

Severe fatigue

You are the nurse caring for a 77-year-old male patient who has been involved in a motor vehicle accident. You and your colleague note that the patients labs indicate minimally elevated serum creatinine levels, which your colleague dismisses. What can this increase in creatinine indicate in older adults?

Substantially reduced renal function

A patient has been living with seasonal allergies for many years, but does not take antihistamines, stating, When I was young I used to take antihistamines, but they always put me to sleep. How should the nurse best respond?

The newer antihistamines are different than in years past, and cause less sedation.

A clinic nurse is providing patient education prior to a patients scheduled palliative radiotherapy to her spine. At the completion of the patient teaching, the patient continues to ask the same questions that the nurse has already addressed. What is the plausible conclusion that the nurse should draw from this?

The patient has not achieved the desired learning outcomes.

A patient with a spinal cord injury is being assessed by the nurse prior to his discharge home from the rehabilitation facility. The nurse is planning care through the lens of the interface model of disability. Within this model, the nurse will plan care based on what belief?

The patient has the potential to function effectively despite his disability.

You are the nurse in a pain clinic caring for an 88-year-old man who is suffering from long-term, intractable pain. At this point, the pain team feels that first-line pharmacological and nonpharmacological methods of pain relief have been ineffective. What recommendation should guide this patients subsequent care?

The patient may benefit from referral to a neurologist or neurosurgeon to discuss pain-management options.

The nurses brief review of a patients electronic health record indicates that the patient regularly undergoes therapeutic phlebotomy. Which of the following rationales for this procedure is most plausible?

The patient may chronically produce excess red blood cells.

A 16-year-old female patient experiences alopecia resulting from chemotherapy, prompting the nursing diagnoses of disturbed body image and situational low self-esteem. What action by the patient would best indicate that she is meeting the goal of improved body image and self-esteem?

The patient requests that her family bring her makeup and wig.

A patient with Von Willebrand disease (vWD) has experienced recent changes in bowel function that suggest the need for a screening colonoscopy. What intervention should be performed in anticipation of this procedure?

The patient should be given necessary clotting factors before the procedure.

A nurse is assessing a patient for risk factors known to contribute to osteoarthritis. What assessment finding would the nurse interpret as a risk factor?

The patients body mass index is 34 (obese).

A community health nurse is performing a visit to the home of a patient who has a history of rheumatoid arthritis (RA). On what aspect of the patients health should the nurse focus most closely during the visit?

The patients functional status

The nurse is providing care for a 90-year-old patient whose severe cognitive and mobility deficits result in the nursing diagnosis of risk for impaired skin integrity due to lack of mobility. When planning relevant assessments, the nurse should prioritize inspection of what area?

The patients heels

A patient who is receiving care for osteosarcoma has been experiencing severe pain since being diagnosed. As a result, the patient has been receiving analgesics on both a scheduled and PRN basis. For the past several hours, however, the patients level of consciousness has declined and she is now unresponsive. How should the patients pain control regimen be affected?

The patients pain control regimen should be continued.

A patient who is scheduled for a skin test informs the nurse that he has been taking corticosteroids to help control his allergy symptoms. What nursing intervention should the nurse implement?

The patients test should be cancelled until he is off his corticosteroids.

An initiative has been launched in a large hospital to promote the use of people-first language in formal and informal communication. What is the significance to the patient when the nurse uses people-first language?

The person is of more importance to the nurse than the disability.

A medical nurse is providing palliative care to a patient with a diagnosis of end-stage chronic obstructive pulmonary disease (COPD). What is the primary goal of this nurses care?

To improve the patients and familys quality of life

A patients rapid cancer metastases have prompted a shift from active treatment to palliative care. When planning this patients care, the nurse should identify what primary aim?

To prevent and relieve suffering

A patient has sought care, stating that she developed hives overnight. The nurses inspection confirms the presence of urticaria. What type of allergic hypersensitivity reaction has the patient developed?

Type I

A patient presents at the dermatology clinic with suspected herpes simplex. The nurse knows to prepare what diagnostic test for this condition?

Tzanck smear

A patient has just died following urosepsis that progressed to septic shock. The patients spouse says, I knew this was coming, but I feel so numb and hollow inside. The nurse should know that these statements are characteristic of what?

Uncomplicated grief and mourning

You have admitted a new patient to your unit with a diagnosis of stage IV breast cancer. This woman has a comorbidity of myasthenia gravis. While you are doing the initial assessment, the patient tells you that she felt the lump in her breast about 9 months ago. You ask the patient why she did not see her health care provider when she first found the lump in her breast. What would be a factor that is known to influence the patient in seeking health care services?

Unfavorable interactions with health care providers

A dermatologist has asked the nurse to assist with examination of a patients skin using a Woods light. This test will allow the physician to assess for which of the following?

Unusual patterns of pigmentation on the patients skin

One aspect of the nurses comprehensive assessment when caring for the terminally ill is the assessment of hope. The nurse is assessing a patient with liver failure for the presence of hope. What would the nurse identify as a hope-fostering category?

Uplifting memories

A 5-year-old boy has been diagnosed with a severe food allergy. What is an important parameter to address when educating the parents of this child about his allergy and care?

Wear a medical identification bracelet.

A nurse is caring for an 87-year-old Mexican-American female patient who is in end-stage renal disease. The physician has just been in to see the patient and her family to tell them that nothing more can be done for the patient and that death is not far. The physician offers to discharge the patient home to hospice care, but the patient and family refuse. After the physician leaves, the patients daughter approaches you and asks what hospice care is. What would this lack of knowledge about hospice care be perceived as?

A barrier to hospice care for this patient

A patient with renal failure has decreased erythropoietin production. Upon analysis of the patients complete blood count, the nurse will expect which of the following results?

A decreased hemoglobin and hematocrit

A 19-year-old patient with a diagnosis of Down syndrome is being admitted to your unit for the treatment of community-acquired pneumonia. When planning this patients care, the nurse recognizes that this patients disability is categorized as what?

A developmental disability

An intensive care nurse is aware of the need to identify patients who may be at risk of developing disseminated intravascular coagulation (DIC). Which of the following ICU patients most likely faces the highest risk of DIC?

A patient who is being treated for septic shock

Which of the following individuals would be the most appropriate candidate for immunotherapy?

A patient with severe allergies to grass and tree pollen

A case manager is responsible for ensuring that patients meet the criteria for diagnoses of chronic conditions in order to ensure their eligibility for federal programs. Which of these definitions may not apply for legal purposes?

A person who is temporarily disabled but later return to full functioning.

A man with a physical disability uses a wheelchair. The individual wants to attend a support group for the parents of autistic children, which is being held in the basement of a church. When the individual arrives at the church, he realizes there are no ramps or elevators to the basement so he will not be able to attend the support group. What type of barrier did this patient encounter?

A structural barrier

A nurse has participated in organizing a blood donation drive at a local community center. Which of the following individuals would most likely be disallowed from donating blood?

A woman whose blood pressure is 88/51 mm Hg

The nurse is preparing to administer a unit of platelets to an adult patient. When administering this blood product, which of the following actions should the nurse perform?

Administer the platelets as rapidly as the patient can tolerate.

A nurse who sits on the hospitals ethics committee is reviewing a complex case that has many of the hallmarks of assisted suicide. Which of the following would be an example of assisted suicide?

Administering a lethal dose of medication to a patient whose death is imminent

Nurses and members of other health disciplines at a states public health division are planning programs for the next 5 years. The group has made the decision to focus on diseases that are experiencing the sharpest increases in their contributions to the overall death rate in the state. This team should plan health promotion and disease prevention activities to address what health problem?

Alzheimers disease

A group of nurses are learning about the high incidence and prevalence of anemia among different populations. Which of the following individuals is most likely to have anemia?

An 81-year-old woman who has chronic heart failure

A clients health history reveals daily consumption of two to three bottles of wine. The nurse should plan assessments and interventions in light of the patients increased risk for what hematologic disorder?

Anemia

A gerontologic nurse is making an effort to address some of the misconceptions about older adults that exist among health care providers. The nurse has made the point that most people aged 75 years remains functionally independent. The nurse should attribute this trend to what factor?

Application of health-promotion and disease-prevention activities

A nurse is doing a shift assessment on a group of patients after first taking report. An elderly patient is having her second dose of IV antibiotics for a diagnosis of pneumonia. The nurse notices a new rash on the patients chest. The nurse should ask what priority question regarding the presence of a reddened rash?

Are you allergic to any foods or medication?

A patient with rheumatoid arthritis comes into the clinic for a routine check-up. On assessment the nurse notes that the patient appears to have lost some of her ability to function since her last office visit. Which of the following is the most appropriate action?

Arrange for the patient to be assessed in her home environment.

A home health nurse makes a home visit to a 90-year-old patient who has cardiovascular disease. During the visit the nurse observes that the patient has begun exhibiting subtle and unprecedented signs of confusion and agitation. What should the home health nurse do?

Arrange for the patient to see his primary care physician.

A patient who is legally blind is being admitted to the hospital. The patient informs the nurse that she needs to have her guide dog present during her hospitalization. What is the nurses best response to the patient?

Arrangements can be made for your guide dog to be at the hospital with you during your stay.

A patient has been living with a diagnosis of anemia for several years and has experienced recent declines in her hemoglobin levels despite active treatment. What assessment finding would signal complications of anemia?

Ascites and peripheral edema

You are caring for a 50-year-old man diagnosed with multiple myeloma; he has just been told by the care team that his prognosis is poor. He is tearful and trying to express his feelings, but he is having difficulty. What should you do first?

Ask if he would like you to sit with him while he collects his thoughts.

The nurse is preparing to insert a peripheral IV catheter into a patient who will require fluids and IV antibiotics. How should the nurse always start the process of insertion?

Ask the patient about allergies to latex or iodine.

A client with several chronic health problems has been newly diagnosed with a qualitative platelet defect. What component of the patients previous medication regimen may have contributed to the development of this disorder?

Aspirin

A child has been transported to the emergency department (ED) after a severe allergic reaction. The ED nurse is evaluating the patients respiratory status. How should the nurse evaluate the patients respiratory status? Select all that apply.

Assess breath sounds. Measure the childs oxygen saturation by oximeter. Monitor the childs respiratory pattern. Assess the childs respiratory rate.

A school nurse is caring for a child who appears to be having an allergic response. What should be the initial action of the school nurse?

Assess for signs and symptoms of anaphylaxis.

A nurse is providing care for a patient who has psoriasis. The nurse is aware of the sequelae that can result from this health problem. Following the appearance of skin lesions, the nurse should prioritize what assessment?

Assessment of the patients joints for pain and decreased range of motion

The community nurse is caring for a patient who has paraplegia following a farm accident when he was an adolescent. This patient is now 64 years old and has just been diagnosed with congestive heart failure. The patient states, Im so afraid about what is going to happen to me. What would be the best nursing intervention for this patient?

Assist the patient in making suitable plans for his care.

The school nurse is teaching a nutrition class in the local high school. One student states that he has heard that certain foods can increase the incidence of cancer. The nurse responds, Research has shown that certain foods indeed appear to increase the risk of cancer. Which of the following menu selections would be the best choice for potentially reducing the risks of cancer?

Baked apricot chicken and steamed broccoli

A woman who is in her third trimester of pregnancy has been experiencing an exacerbation of iron-deficiency anemia in recent weeks. When providing the patient with nutritional guidelines and meal suggestions, what foods would be most likely to increase the womans iron stores?

Beef liver accompanied by orange juice

A nurse is performing the initial assessment of a patient who has a recent diagnosis of systemic lupus erythematosus (SLE). What skin manifestation would the nurse expect to observe on inspection?

Butterfly rash

The nurse is performing a comprehensive assessment of a patients skin surfaces and intends to assess moisture, temperature, and texture. The nurse should perform this component of assessment in what way?

By palpating the patients skin

A gerontologic nurse is teaching a group of nursing students about integumentary changes that occur in older adults. How should these students best integrate these changes into care planning?

By protecting older adults against shearing injuries

Atazanavir (ATV, Reyataz) requires what priority intervention?

Cardiac assessment

The hospice nurse has just admitted a new patient to the program. What principle guides hospice care?

Care addresses the needs of the patient as well as the needs of the family.

A nurse is caring for a patient undergoing rehabilitation following a snowboarding accident. Within the interdisciplinary team, the nurse has been given the responsibility for coordinating the patients total rehabilitative plan of care. What nursing role is this nurse performing?

Case manager

A patient has just been diagnosed with psoriasis and frequently has lesions around his right eye. What should the nurse teach the patient about topical corticosteroid use on these lesions?

Cataract development is possible.

A patient tells the nurse that her doctor just told her that her new diagnosis of rheumatoid arthritis is considered to be a chronic condition. She asks the nurse what chronic condition means. What would be the nurses best response?

Chronic conditions are defined as health problems that require management of several months or longer.

You are the nurse coming on shift in a rehabilitation unit. You receive information in report about a new patient who has fibromyalgia and has difficulty with her ADLs. The off-going nurse also reports that the patient is withdrawn, refusing visitors, and has been vacillating between tears and anger all afternoon. What do you know about chronic pain syndromes that could account for your new patients behavior?

Chronic pain can cause intense emotional responses

The nurse is caring for a patient in metabolic alkalosis. The patient has an NG tube to low intermittent suction for a diagnosis of bowel obstruction. What drug would the nurse expect to find on the medication orders?

Cimetidine

A 65-year-old man presents at the clinic complaining of nodules on both legs. The man tells the nurse that his son, who is in medical school, encouraged him to seek prompt care and told him that the nodules are related to the fact that he is Jewish. What health problem should the nurse suspect?

Classic Kaposis sarcoma

A nurse educator is reviewing peripheral IV insertion with a group of novice nurses. How should these nurses be encouraged to deal with excess hair at the intended site?

Clip the hair in the area.

A patient is being treated for DIC and the nurse has prioritized the nursing diagnosis of Risk for Deficient Fluid Volume Related to Bleeding. How can the nurse best determine if goals of care relating to this diagnosis are being met?

Closely monitor intake and output.

You are the nurse caring for patients in the urology clinic. A new patient, 78 years old, presents with complaints of urinary incontinence. An anticholinergic is prescribed. Why might this type of medication be an inappropriate choice in the elderly population?

Confusion can be an adverse effect of this medication.

The physician has ordered a peripheral IV to be inserted before the patient goes for computed tomography. What should the nurse do when selecting a site on the hand or arm for insertion of an IV catheter?

Consider potential effects on the patients mobility when selecting a site.

The hospice nurse is caring for a patient with cancer in her home. The nurse has explained to the patient and the family that the patient is at risk for hypercalcemia and has educated them on that signs and symptoms of this health problem. What else should the nurse teach this patient and family to do to reduce the patients risk of hypercalcemia?

Consume 2 to 4 L of fluid daily.

A night nurse is reviewing the next days medication administration record (MAR) of a patient who has hemophilia. The nurse notes that the MAR specifies both oral and subcutaneous options for the administration of a PRN antiemetic. What is the nurses best action?

Contact the prescriber to have the subcutaneous option discontinued.

The nurse in an allergy clinic is educating a new patient about the pathology of the patients health problem. What response should the nurse describe as a possible consequence of histamine release?

Contraction of bronchial smooth muscle

The nurse is assessing a 73-year-old patient who was diagnosed with metastatic prostate cancer. The nurse notes that the patient is exhibiting signs of loss, grief, and intense sadness. Based on this assessment data, the nurse will document that the patient is most likely in what stage of death and dying?

Depression

The nurse is caring for a 51-year-old female patient whose medical history includes chronic fatigue and poorly controlled back pain. These medical diagnoses should alert the nurse to the possibility of what consequent health problem?

Depression

A major cause of health-related problems is the increase in the incidence of chronic conditions. This is the case not only in developed countries like the United States but also in developing countries. What factor has contributed to the increased incidence of chronic diseases in developing countries?

Developing countries are simultaneously coping with emerging infectious diseases.

A 54-year-old has a diagnosis of breast cancer and is tearfully discussing her diagnosis with the nurse. The patient states, They tell me my cancer is malignant, while my coworkers breast tumor was benign. I just dont understand at all. When preparing a response to this patient, the nurse should be cognizant of what characteristic that distinguishes malignant cells from benign cells of the same tissue type?

Different proteins in the cell membrane

A patient on the medical unit is receiving a unit of PRBCs. Difficult IV access has necessitated a slow infusion rate and the nurse notes that the infusion began 4 hours ago. What is the nurses most appropriate action?

Discontinue the remainder of the PRBC transfusion and inform the physician.

A patient is receiving the first of two ordered units of PRBCs. Shortly after the initiation of the transfusion, the patient complains of chills and experiences a sharp increase in temperature. What is the nurses priority action?

Discontinue the transfusion.

The nurse is caring for a patient has just been given a 6-month prognosis following a diagnosis of extensive stage small-cell lung cancer. The patient states that he would like to die at home, but the team believes that the patients care needs are unable to be met in a home environment. What might you suggest as an alternative?

Discuss a referral for hospice care.

A nurse is planning the care of a patient who has been diagnosed with renal failure, which the nurse recognizes as being a chronic condition. Which of the following descriptors apply to chronic conditions? Select all that apply.

Diseases where complete cures are rare Diseases that do not resolve spontaneously Diseases that have a prolonged course

The nurse is performing an initial assessment of a patient who has a raised, pruritic rash. The patient denies taking any prescription medication and denies any allergies. What would be an appropriate question to ask this patient at this time?

Do you take any over-the-counter drugs or herbal preparations?

A nurse is conducting a health interview and is assessing for integumentary conditions that are known to have a genetic component. What assessment question is most appropriate?

Does anyone in your family have eczema or psoriasis?

You are caring for a patient, a 42-year-old mother of two children, with a diagnosis of ovarian cancer. She has just been told that her ovarian cancer is terminal. When you admitted this patient, you did a spiritual assessment. What question would it have been most important for you to evaluate during this assessment?

Does she have a sense of peace of mind and a purpose to her life?

The nurse caring for a 91-year-old patient with osteoarthritis is reviewing the patients chart. This patient is on a variety of medications prescribed by different care providers in the community. In light of the QSEN competency of safety, what is the nurse most concerned about with this patient?

Drug interactions

A child is undergoing testing for food allergies after experiencing unexplained signs and symptoms of hypersensitivity. What food items would the nurse inform the parents are common allergens?

Eggs and wheat

A clinic nurse is caring for a patient diagnosed with rheumatoid arthritis (RA). The patient tells the nurse that she has not been taking her medication because she usually cannot remove the childproof medication lids. How can the nurse best facilitate the patients adherence to her medication regimen?

Encourage her to have her pharmacy replace the tops with alternatives that are easier to open.

A nurse is caring for a patient who has sickle cell anemia and the nurses assessment reveals the possibility of substance abuse. What is the nurses most appropriate action?

Encourage the patient to seek care from a single provider for pain relief.

Two patients on your unit have recently returned to the postsurgical unit after knee arthroplasty. One patient is reporting pain of 8 to 9 on a 0-to-10 pain scale, whereas the other patient is reporting a pain level of 3 to 4 on the same pain scale. What is the nurses most plausible rationale for understanding the patients different perceptions of pain?

Endorphin levels may vary between patients, affecting the perception of pain.

An adult patient has been diagnosed with iron-deficiency anemia. What nursing diagnosis is most likely to apply to this patients health status?

Fatigue related to decreased oxygen-carrying capacity

A nurse who provides care on an acute medical unit has observed that physicians are frequently reluctant to refer patients to hospice care. What are contributing factors that are known to underlie this tendency? Select all that apply.

Financial pressures on health care providers Patient reluctance to accept this type of care Advances in curative treatment in late-stage illness

You are caring for a patient who has a diagnosis of syndrome of inappropriate antidiuretic hormone secretion (SIADH). Your patients plan of care includes assessment of specific gravity every 4 hours. The results of this test will allow the nurse to assess what aspect of the patients health?

Fluid volume status

A patient has been brought to the emergency department by EMS after being found unresponsive. Rapid assessment reveals anaphylaxis as a potential cause of the patients condition. The care team should attempt to assess for what potential causes of anaphylaxis? Select all that apply.

Foods Medications Insect stings

One day after a patient is admitted to the medical unit, you note that the patient is oliguric. You notify the acute-care nurse practitioner who orders a fluid challenge of 200 mL of normal saline solution over 15 minutes. This intervention will achieve which of the following?

Help distinguish reduced renal blood flow from decreased renal function

The nurse is describing normal RBC physiology to a patient who has a diagnosis of anemia. The nurse should explain that the RBCs consist primarily of which of the following?

Hemoglobin

The results of a patients most recent blood work and physical assessment are suggestive of immune thrombocytopenic purpura (ITP). This patient should undergo testing for which of the following potential causes? Select all that apply.

Hepatitis HIV

A 52-year-old married man with two adolescent children is beginning rehabilitation following a motor vehicle accident. You are the nurse planning the patients care. Who will the patients condition affect?

Him and his entire family

You are caring for a young woman who has Down syndrome and who has just been diagnosed with type 2 diabetes. What consideration should you prioritize when planning this patients nursing care?

How her developmental disability influences her health management

You are caring for a patient admitted to the medical-surgical unit after falling from a horse. The patient states I hurt so bad. I suffer from chronic pain anyway, and now it is so much worse. When planning the patients care, what variables should you consider? Select all that apply.

How the presence of pain affects patients and families Resources that can assist the patient with pain management The advantages and disadvantages of available pain-relief strategies

When planning the care of a patient with a fluid imbalance, the nurse understands that in the human body, water and electrolytes move from the arterial capillary bed to the interstitial fluid. What causes this to occur?

Hydrostatic pressure resulting from the pumping action of the heart

You are caring for a patient who is being treated on the oncology unit with a diagnosis of lung cancer with bone metastases. During your assessment, you note the patient complains of a new onset of weakness with abdominal pain. Further assessment suggests that the patient likely has a fluid volume deficit. You should recognize that this patient may be experiencing what electrolyte imbalance?

Hypercalcemia

You are called to your patients room by a family member who voices concern about the patients status. On assessment, you find the patient tachypnic, lethargic, weak, and exhibiting a diminished cognitive ability. You also find 3+ pitting edema. What electrolyte imbalance is the most plausible cause of this patients signs and symptoms?

Hyperchloremia

You are the surgical nurse caring for a 65-year-old female patient who is postoperative day 1 following a thyroidectomy. During your shift assessment, the patient complains of tingling in her lips and fingers. She tells you that she has an intermittent spasm in her wrist and hand and she exhibits increased muscle tone. What electrolyte imbalance should you first suspect?

Hypocalcemia

A nurse is working with a patient with rheumatic disease who is being treated with salicylate therapy. What statement would indicate that the patient is experiencing adverse effects of this drug?

I have this ringing in my ears that just wont go away.

A patient has presented with signs and symptoms that are consistent with contact dermatitis. What aspect of care should the nurse prioritize when working with this patient?

Identifying the offending agent, if possible

The nurse is reviewing the importance of preventative health care with a patient who has a disability. The patient states that she will not have the money to pay for her annual gynecologic exams or mammograms due to the cost of this hospitalization. What information would be appropriate for the nurse to share with the patient?

Ill look into federal assistance programs that provide financial assistance for health-related expenses for people with disabling conditions.

A patient with systemic lupus erythematosus (SLE) is preparing for discharge. The nurse knows that the patient has understood health education when the patient makes what statement?

Ill make sure to monitor my body temperature on a regular basis.

The nurse is caring for a patient who is to begin receiving external radiation for a malignant tumor of the neck. While providing patient education, what potential adverse effects should the nurse discuss with the patient?

Impaired nutritional status

While assessing a newly admitted patient you note the following: impaired coordination, decreased muscle strength, limited range of motion, and reluctance to move. What nursing diagnosis do these signs and symptoms most clearly suggest?

Impaired physical mobility

A nurse is a long-term care facility is admitting a new resident who has a bleeding disorder. When planning this residents care, the nurse should include which of the following?

Implementing of a plan for fall prevention

A patient with a hematologic disorder asks the nurse how the body forms blood cells. The nurse should describe a process that takes place where?

In the bone marrow

You are the case manager who oversees the multidisciplinary care of several patients living with chronic conditions. Two of your patients are living with spina bifida. You recognize that the center of care for these two patients typically exists where?

In the home

A patient has been admitted to the emergency department with signs of anaphylaxis following a bee sting. The nurse knows that if this is a true allergic reaction the patient will present with what alteration in laboratory values?

Increased eosinophils

A nurse is aware that the number of people in the United States who are living with disabilities is expected to continue increasing. What is considered to be one of the factors contributing to this increase?

Increased survival rates among people who experience trauma

An 82-year-old patient is being treated in the hospital for a sacral pressure ulcer. What age-related change is most likely to affect the patients course of treatment?

Increased time required for wound healing

A clinic nurse is caring for a patient with suspected gout. While explaining the pathophysiology of gout to the patient, the nurse should describe which of the following?

Increased uric acid levels

A nurse is giving a talk to a local community group whose members advocate for disabled members of the community. The group is interested in emerging trends that are impacting the care of people who are disabled in the community. The nurse should describe an increasing focus on what aspect of care?

Independent living

Older people have many altered reactions to disease that are based on age-related physiological changes. When the nurse observes physical indicators of illness in the older population, that nurse must remember which of the following principles?

Indicators that are useful and reliable in younger populations cannot be relied on as indications of potential life-threatening problems in older adults.

The nurse is providing care for an older adult man whose diagnosis of dementia has recently led to urinary incontinence. When planning this patients care, what intervention should the nurse avoid?

Indwelling catheter

A patient with SLE has come to the clinic for a routine check-up. When auscultating the patients apical heart rate, the nurse notes the presence of a distinct scratching sound. What is the nurses most appropriate action?

Inform the primary care provider that a friction rub may be present.

A patient is being treated on the medical unit for a sickle cell crisis. The nurses most recent assessment reveals an oral temperature of 100.5F and a new onset of fine crackles on lung auscultation. What is the nurses most appropriate action?

Inform the primary care provider that the patient may have an infection.

A 74-year-old woman was diagnosed with rheumatoid arthritis 1 year ago, but has achieved adequate symptom control through the regular use of celecoxib (Celebrex), a COX-2 selective NSAID. The nurse should recognize that this drug, like other NSAIDs, influences what aspect of the pathophysiology of nociceptive pain?

Inhibiting transduction by blocking the formation of prostaglandins in the periphery

An oncology patient will begin a course of chemotherapy and radiation therapy for the treatment of bone metastases. What is one means by which malignant disease processes transfer cells from one place to another?

Invading healthy host tissues

A patient has come to the OB/GYN clinic due to recent heavy menstrual flow. Because of the patients consequent increase in RBC production, the nurse knows that the patient may need to increase her daily intake of what substance?

Iron

A patient comes to the clinic complaining of fatigue and the health interview is suggestive of pica. Laboratory findings reveal a low serum iron level and a low ferritin level. With what would the nurse suspect that the patient will be diagnosed?

Iron deficiency anemia

A nurse is providing education to a patient with iron deficiency anemia who has been prescribed iron supplements. What should the nurse include in health education?

Iron will cause the stools to darken in color.

A 74-year-old woman experienced a cerebrovascular accident 6 weeks ago and is currently receiving inpatient rehabilitation. You are coaching the patient to contract and relax her muscles while keeping her extremity in a fixed position. Which type of exercise is the patient performing?

Isometric

The organization of a patients care on the palliative care unit is based on interdisciplinary collaboration. How does interdisciplinary collaboration differ from multidisciplinary practice?

It is based on communication and cooperation between disciplines.

You are part of the health care team caring for an 87-year-old woman who has been admitted to your rehabilitation facility after falling and fracturing her left hip. The patient appears to be failing to regain functional ability and may have to be readmitted to an acute-care facility. When planning this patients care, what do you know about the negative effects of the stress associated with pain?

It is particularly harmful in the elderly who have been injured or who are ill.

The community health nurse is performing a home visit to an 84-year-old woman recovering from hip surgery. The nurse notes that the woman seems uncharacteristically confused and has dry mucous membranes. When asked about her fluid intake, the patient states, I stop drinking water early in the day because it is just too difficult to get up during the night to go to the bathroom. What would be the nurses best response?

Limiting your fluids can create imbalances in your body that can result in confusion. Maybe we need to adjust the timing of your fluids.

A nurse who works in an oncology clinic is assessing a patient who has arrived for a 2-month follow-up appointment following chemotherapy. The nurse notes that the patients skin appears yellow. Which blood tests should be done to further explore this clinical sign?

Liver function tests (LFTs)

You are admitting a patient to your rehabilitation unit who has a diagnosis of persistent, severe pain. According to the patients history, the patients pain has not responded to conventional approaches to pain management. What treatment would you expect might be tried with this patient?

Long-term intrathecal or epidural catheter

You are providing care for an 82-year-old man whose signs and symptoms of Parkinson disease have become more severe over the past several months. The man tells you that he can no longer do as many things for himself as he used to be able to do. What factor should you recognize as impacting your patients life most significantly?

Loss of independence

You are caring for a patient with a diagnosis of pancreatitis. The patient was admitted from a homeless shelter and is a vague historian. The patient appears malnourished and on day 3 of the patients admission total parenteral nutrition (TPN) has been started. Why would you know to start the infusion of TPN slowly?

Malnourished patients receiving parenteral nutrition are at risk for hypophosphatemia if calories are started too aggressively.

A nurse is creating a teaching plan for a patient who has a recent diagnosis of scleroderma. What topics should the nurse address during health education? Select all that apply.

Managing Raynauds-type symptoms Smoking cessation The importance of vigilant skin care

The nurse is providing health education to the parents of a toddler who has been diagnosed with food allergies. What should the nurse teach this family about the childs health problem?

Many children outgrow their food allergies in a few years if they avoid the offending foods.

You are planning rehabilitation activities for a patient who is working toward discharge back into the community. During a care conference, the team has identified a need to focus on the patients instrumental activities of daily living (IADLs). When planning the patients subsequent care, you should focus particularly on which of the following?

Meal preparation

A patient with end-stage heart failure has participated in a family meeting with the interdisciplinary team and opted for hospice care. On what belief should the patients care in this setting be based?

Meaningful living during terminal illness is best supported in the home.

The nurse is caring for a young adult male with a traumatic brain injury and severe disabilities caused by a motor vehicle accident when he was an adolescent. Where does the nurse often provide care for patients like this young adult?

Medical-surgical units

An elderly patient, while being seen in an urgent care facility for a possible respiratory infection, asks the nurse if Medicare is going to cover the cost of the visit. What information can the nurse give the patient to help allay her concerns?

Medicare has a copayment for many of the services it covers. This requires the patient to pay a part of the bill

Falls, which are a major health problem in the elderly population, occur from multifactorial causes. When implementing a comprehensive plan to reduce the incidence of falls on a geriatric unit, what risk factors should nurses identify? Select all that apply.

Medication effects Poor lighting Sensory impairment

A nurse is providing an educational presentation addressing the topic of Protecting Your Skin. When discussing the anatomy of the skin with this group, the nurse should know that what cells are responsible for producing the pigmentation of the skin?

Melanocytes

You are an emergency-room nurse caring for a trauma patient. Your patient has the following arterial blood gas results: pH 7.26, PaCO2 28, HCO3 11 mEq/L. How would you interpret these results?

Metabolic acidosis with a compensatory respiratory alkalosis

You are caring for a 65-year-old male patient admitted to your medical unit 72 hours ago with pyloric stenosis. A nasogastric tube placed upon admission has been on low intermittent suction ever since. Upon review of the mornings blood work, you notice that the patients potassium is below reference range. You should recognize that the patient may be at risk for what imbalance?

Metabolic alkalosis

You are frequently assessing an 84-year-old womans pain after she suffered a humeral fracture in a fall. When applying the nursing process in pain management for a patient of this age, what principle should you best apply?

Monitor for signs of drug toxicity due to a decrease in metabolism.

A patient has been scheduled for a bone marrow biopsy and admits to the nurse that she is worried about the pain involved with the procedure. What patient education is most accurate?

Most people feel some brief, sharp pain when the needle enters the bone.

The nurse is caring for a patient who has terminal lung cancer and is unconscious. Which assessment finding would most clearly indicate to the nurse that the patients death is imminent?

Mottling of the lower limbs

The nurse is accepting care of an adult patient who has been experiencing severe and intractable pain. When reviewing the patients medication administration record, the nurse notes the presence of gabapentin (Neurontin). The nurse is justified in suspecting what phenomenon in the etiology of the patients pain?

Neuropathy

A client cuts her hand on a window. Which cell type would be elevated first in an attempt to prevent infection in the clients hand?

Neutrophils

A patient with end-stage lung cancer has been admitted to hospice care. The hospice team is meeting with the patient and her family to establish goals for care. What is likely to be a first priority in goal setting for the patient?

Pain control

You are caring for a patient who has just been told that her stage IV colon cancer has recurred and metastasized to the liver. The oncologist offers the patient the option of surgery to treat the progression of this disease. What type of surgery does the oncologist offer?

Palliative

A nurse is preparing to perform the physical assessment of a newly admitted patient. During which of the following components of the assessment should the nurse wear gloves? Select all that apply.

Palpation of a rash on the patients trunk Palpation of a lesion on the patients upper back

A patient is undergoing rehabilitation following a stroke that left him with severe motor and sensory deficits. The patient has been unable to ambulate since his accident, but has recently achieved the goals of sitting and standing balance. What is the patient now able to use?

Parallel bars

You are the nurse caring for a female patient who developed a pressure ulcer as a result of decreased mobility. The nurse on the shift before you has provided patient teaching about pressure ulcers and healing promotion. You assess that the patient has understood the teaching by observing what?

Patient avoids placing her body weight on the healing site.

You are caring for a 20-year-old patient with a diagnosis of cerebral palsy who has been admitted for the relief of painful contractures in his lower extremities. When creating a nursing care plan for this patient, what variables should the nurse consider? Select all that apply.

Patients comorbid conditions Type of procedure be performed Changes in neurologic function due to the procedure Prior effectiveness in relieving the pain

During their prime employable years between ages 21 and 64, 77% of those with a nonsevere disability are employed. What has research shown about this employed population?

People with disabilities earn less money than people without disabilities.

A female patient, 47 years old, visits the clinic because she has been experiencing stress incontinence when she sneezes or exercises vigorously. What is the best instruction the nurse can give the patient?

Perform Kegel exercises four to six times per day.

A patient diagnosed with a stasis ulcer has been hospitalized. There is an order to change the dressing and provide wound care. Which activity should the nurse first perform when providing wound care?

Perform hand hygiene.

A young man with a diagnosis of hemophilia A has been brought to emergency department after suffering a workplace accident resulting in bleeding. Rapid assessment has revealed the source of the patients bleeding and established that his vital signs are stable. What should be the nurses next action?

Prepare for the administration of factor VIII.

A nurses plan of care for a patient with rheumatoid arthritis includes several exercise-based interventions. Exercises for patients with rheumatoid disorders should have which of the following goals?

Preserve and increase range of motion while limiting joint stress

The nurse is caring for a male patient whose diagnosis of bone cancer is causing severe and increasing pain. Before introducing nonpharmacological pain control interventions into the patients plan of care, the nurse should teach the patient which of the following?

Reducing his use of analgesics is not the purpose of these interventions.

A patient was prescribed an oral antibiotic for the treatment of sinusitis. The patient has now stopped, stating she developed a rash shortly after taking the first dose of the drug. What is the nurses most appropriate response?

Refer the woman to her primary care provider to have the medication changed.

A patient who is receiving rehabilitation following a spinal cord injury has been diagnosed with reflex incontinence. The nurse caring for the patient should include which intervention in this patients plan of care?

Regular perineal care to prevent skin breakdown

A patient who is recovering from a stroke expresses frustration about his care to the nurse, stating, It seems like everyone sees me as just a problem that needs fixing. This patients statement is suggestive of what model of disability?

Rehabilitation model

The nurse is assessing a patients pain while the patient awaits a cholecystectomy. The patient is tearful, hesitant to move, and grimacing. When asked, the patient rates his pain as a 2 at this time using a 0-to-10 pain scale. How should the nurse best respond to this assessment finding?

Reinforce teaching about the pain scale number system.

A medical nurse is providing end-of-life care for a patient with metastatic bone cancer. The nurse notes that the patient has been receiving oral analgesics for her pain with adequate effect, but is now having difficulty swallowing the medication. What should the nurse do?

Request the physician to order analgesics by an alternative route.

The clinic nurse is caring for a patient whose grandmother and sister have both had breast cancer. She requested a screening test to determine her risk of developing breast cancer and it has come back positive. The patient asks you what she can do to help prevent breast cancer from occurring. What would be your best response?

Research has shown that taking the drug tamoxifen can reduce your chance of breast cancer.

As the American population ages, nurses expect see more patients admitted to long-term care facilities in need of palliative care. Regulations now in place that govern how the care in these facilities is both organized and reimbursed emphasize what aspect of care?

Restorative measures

A patient with rheumatoid arthritis comes to the clinic complaining of pain in the joint of his right great toe and is eventually diagnosed with gout. When planning teaching for this patient, what management technique should the nurse emphasize?

Restrict consumption of foods high in purines.

A nurse is caring for a patient admitted to the medical unit with a diagnosis of pemphigus vulgaris. When writing the care plan for this patient, what nursing diagnoses should be included? Select all that apply.

Risk for Infection Related to Lesions Impaired Skin Integrity Related to Epidermal Blisters Disturbed Body Image Related to Presence of Skin Lesions Acute Pain Related to Disruption in Skin Integrity

Mrs. Harris is an 83-year-old woman who has returned to the community following knee replacement surgery. The community health nurse recognizes that Mrs. Harris has prescriptions for nine different medications for the treatment of varied health problems. In addition, she has experienced occasional episodes of dizziness and lightheadedness since her discharge. The nurse should identify which of the following nursing diagnoses?

Risk for falls related to polypharmacy and impaired balance

A 62-year-old woman diagnosed with breast cancer is scheduled for a partial mastectomy. The oncology nurse explained that the surgeon will want to take tissue samples to ensure the disease has not spread to adjacent axillary lymph nodes. The patient has asked if she will have her lymph nodes dissected, like her mother did several years ago. What alternative to lymph node dissection will this patient most likely undergo?

Sentinel node biopsy

A patient is receiving a blood transfusion and complains of a new onset of slight dyspnea. The nurses rapid assessment reveals bilateral lung crackles and elevated BP. What is the nurses most appropriate action?

Slow the infusion rate and monitor the patient closely.

For several years, a community health nurse has been working with a 78-year-old man who requires a wheelchair for mobility. The nurse is aware that the interactions between disabilities and aging are not yet clearly understood. This interaction varies, depending on what variable?

Socioeconomics

A wound care nurse is reviewing skin anatomy with a group of medical nurses. Which area of the skin would the nurse identify as providing a cushion between the skin layers, muscles, and bones?

Subcutaneous tissue

A school nurse is providing health promotion teaching to a group of high school seniors. The nurse should highlight what salient risk factor for traumatic brain injury?

Substance abuse

A nurse is admitting a patient with immune thrombocytopenic purpura to the unit. In completing the admission assessment, the nurse must be alert for what medications that potentially alter platelet function? Select all that apply.

Sulfa-containing medications Aspirin-based drugs NSAIDs

A patient who has sustained third-degree facial burns and a facial fracture is undergoing reconstructive surgery and implantation of a prosthesis. The nurse has identified a nursing diagnosis of Disturbed Body Image Related to Disfigurement. What would be an appropriate nursing intervention related to this diagnosis?

Teaching the patient how to use and care for the prosthesis

The nurse caring for a 79-year-old man who has just returned to the medicalsurgical unit following surgery for a total knee replacement received report from the PACU. Part of the report had been passed on from the preoperative assessment where it was noted that he has been agitated in the past following opioid administration. What principle should guide the nurses management of the patients pain?

The elderly may require lower doses of medication and are easily confused with new medications.

A patients electronic health record notes that he has previously undergone treatment for secondary polycythemia. How should this aspect of the patients history guide the nurses subsequent assessment?

The nurse should assess for evidence of lung disease.

Which of the following circumstances would most clearly warrant autologous blood donation?

The patient has elective surgery pending.

A patient is being treated for the effects of a longstanding vitamin B12 deficiency. What aspect of the patients health history would most likely predispose her to this deficiency?

The patient is a vegan.

The nurse is caring for a patient who has been recently diagnosed with late stage pancreatic cancer. The patient refuses to accept the diagnosis and refuses to adhere to treatment. What is the most likely psychosocial purpose of this patients strategy?

The patient may be trying to protect loved ones from the emotional effects of the illness.

A patient presents at the free clinic with a black, wart-like lesion on his face, stating, Ive done some research, and Im pretty sure I have malignant melanoma. Subsequent diagnostic testing results in a diagnosis of seborrheic keratosis. The nurse should recognize what significance of this diagnosis?

The patient requires no treatment unless he finds the lesion to be cosmetically unacceptable.

A patient with severe environmental allergies is scheduled for an immunotherapy injection. What should be included in teaching the patient about this treatment?

The patient will remain in the clinic to be monitored for 30 minutes following the injection.

The staff development nurse is presenting a class on the importance of incorporating people-first language into daily practice as well as documentation. What is an example of the use of people-first language when giving a verbal report?

The patient with schizophrenia

A patient with a pulmonary embolism is being treated with a heparin infusion. What diagnostic finding suggests to the nurse that treatment is effective?

The patients activated partial thromboplastin time (aPTT) is 1.5 to 2.5 times the control value.

Your patient has just returned from the postanesthetic care unit (PACU) following left tibia open reduction internal fixation (ORIF). The patient is complaining of pain, and you are preparing to administer the patients first scheduled dose of hydromorphone (Dilaudid). Prior to administering the drug, you would prioritize which of the following assessments?

The patients allergy status

A patients decline in respiratory and renal function has been attributed to Goodpasture syndrome, which is a type II hypersensitivity reaction. What pathologic process underlies the patients health problem?

The patients body has mistakenly identified a normal constituent of the body as foreign.

The nurse is part of the health care team at an oncology center. A patient has been diagnosed with leukemia and the prognosis is poor, but the patient is not yet aware of the prognosis. How can the bad news best be conveyed to the patient?

The prognosis should be delivered with the patient at eye level.

You are the nurse caring for a young mother who has a longstanding diagnosis of multiple sclerosis (MS). She was admitted to your unit with a postpartum infection 3 days ago. You are planning to discharge her home when she has finished 5 days of IV antibiotic therapy. With what information would it be most important for you to provide this patient?

The same information you would provide to a patient without a chronic condition

You are presenting patient teaching to a 48-year-old man who was just diagnosed with type 2 diabetes. The patient has a BMI of 35 and leads a sedentary lifestyle. You give the patient information on the risk factors for his diagnosis and begin talking with him about changing behaviors around diet and exercise. You know that further patient teaching is necessary when your patient tells you what?

There is nothing that can be done anyway, because chronic diseases like diabetes cannot be prevented.

The clinic nurse is caring for a 42-year-old male oncology patient. He complains of extreme fatigue and weakness after his first week of radiation therapy. Which response by the nurse would best reassure this patient?

These symptoms usually result from radiation therapy; however, we will continue to monitor your laboratory and x-ray studies.

You are creating a nursing care plan for a patient with a primary diagnosis of cellulitis and a secondary diagnosis of chronic pain. What common trait of patients who live with chronic pain should inform your care planning?

They can experience acute pain in addition to chronic pain.

A nurse is caring for a teenage girl who has had an anaphylactic reaction after a bee sting. The nurse is providing patient teaching prior to the patients discharge. In the event of an anaphylactic reaction, the nurse informs the patient that she should self-administer epinephrine in what site?

Thigh

A patient has just been told that her illness is terminal. The patient tearfully states, I cant believe I am going to die. Why me? What is your best response?

This must be very difficult for you.

Assessment of a patients leg reveals the presence of a 1.5-cm circular region of necrotic tissue that is deeper than the epidermis. The nurse should document the presence of what type of skin lesion?

Ulcer

An unlicensed nursing assistant (NA) reports to the nurse that a postsurgical patient is complaining of pain that she rates as 8 on a 0-to-10 point scale. The NA tells the nurse that he thinks the patient is exaggerating and does not need pain medication. What is the nurses best response?

Unless there is strong evidence to the contrary, we should take the patients report at face value.

You are caring for a patient with sickle cell disease in her home. Over the years, there has been joint damage, and the patient is in chronic pain. The patient has developed a tolerance to her usual pain medication. When does the tolerance to pain medication become the most significant problem?

When it results in inadequate relief from pain

A patient with an exacerbation of systemic lupus erythematosus (SLE) has been hospitalized on the medical unit. The nurse observes that the patient expresses angerand irritation when her call bell isnt answered immediately. What would be the most appropriate response?

You seem like youre feeling angry. Is that something that we could talk about?

Effective teaching statement regarding immunodeficiencies

"The majority of primary immunodeficiencies are diagnosed in infancy."

HIV therapy teaching

-Antiretroviral therapy targets different stages of the HIV life cycle -The CD4 count is the major indicator of immune function and guides therapy

Patient with CVID has an order for an IVIG infusion, What actions should the nurse perform prior to infusion?

-Assess baseline vitals -Assess for hx of migraine headaches -Obtain pt height and weight -Premedicate with acetaminophen and diphenhydramine

The nurse is caring for a 65-year-old patient who has previously been diagnosed with hypertension. Which of the following blood pressure readings represents the threshold between high-normal blood pressure and hypertension?

140/90 mm Hg

Antiretroviral medications should be offered to individuals with T-cell counts of less than

350 cell/mm

Research has corroborated an experienced nurses observation that the incidence and prevalence of chronic conditions is increasing in the United States. What health promotion initiative most directly addresses the factor that has been shown to contribute to this increase?

A community-based weight-loss program

You are the nurse planning an educational event for the nurses on a subacute medical unit on the topic of normal, age-related physiological changes. What phenomenon would you include in your teaching plan?

A decrease in muscle mass and bone density

A gerontologic nurse is aware of the demographic changes that are occurring in the United States, and this affects the way that the nurse plans and provides care. Which of the following phenomena is currently undergoing the most rapid and profound change?

A growing number of people live to a very old age.

A patient comes into the clinic complaining of fatigue. Blood work shows an increased bilirubin concentration and an increased reticulocyte count. What would the nurse suspect the patient has?

A hemolytic anemia

While assessing a 25-year-old female, the nurse notes that the patient has hair on her lower abdomen. Earlier in the health interview, the patient stated that her menses are irregular. The nurse should suspect what type of health problem?

A hormonal imbalance

A nurse knows of several patients who have achieved adequate control of their allergy symptoms using over-the-counter antihistamines. Antihistamines would be contraindicated in the care of which patient?

A pregnant woman at 30 weeks gestation

A patient with human immunodeficiency virus (HIV) has sought care because of the recent development of new skin lesions. The nurse should interpret these lesions as most likely suggestive of what?

A reduction in the patients CD4 count

A nurse who works in the specialty of palliative care frequently encounters issues and situations that constitute ethical dilemmas. What issue has most often presented challenging ethical issues, especially in the context of palliative care?

Ability of technology to prolong life beyond meaningful quality of life

You are the emergency department (ED) nurse caring for an adult patient who was in a motor vehicle accident. Radiography reveals an ulnar fracture. What type of pain are you addressing when you provide care for this patient?

Acute

You are the nurse caring for an elderly patient who is being treated for community-acquired pneumonia. Since the time of admission, the patient has been disoriented and agitated to varying degrees. Appropriate referrals were made and the patient was subsequently diagnosed with dementia. What nursing diagnosis should the nurse prioritize when planning this patients care?

Acute confusion related to dementia

A patient who has recently been diagnosed with chronic heart failure is being taught by the nurse how to live successfully with her chronic condition. Her ability to meet this goal will primarily depend on her ability to do which of the following?

Adapt her lifestyle to accommodate her symptoms.

An elderly woman diagnosed with osteoarthritis has been referred for care. The patient has difficulty ambulating because of chronic pain. When creating a nursing care plan, what intervention may the nurse use to best promote the patients mobility?

Administer an analgesic as ordered to facilitate the patients mobility.

A 50-year-old man diagnosed with leukemia will begin chemotherapy. What would the nurse do to combat the most common adverse effects of chemotherapy?

Administer an antiemetic.

A 66-year-old patient is in a hospice receiving palliative care for lung cancer which has metastasized to the patients liver and bones. For the past several hours, the patient has been experiencing dyspnea. What nursing action is most appropriate to help to relive the dyspnea the patient is experiencing?

Administer bronchodilators and corticosteroids, as ordered.

A patient being treated for HIV/AIDS has decreased appetite, almost to the point of anorexia. What is the nurse's best action?

Administer megestrol acetate (Megace)

The admissions department at a local hospital is registering an elderly man for an outpatient diagnostic test. The admissions nurse asks the man if he has an advanced directive. The man responds that he does not want to complete an advance directive because he does not want anyone controlling his finances. What would be appropriate information for the nurse to share with this patient?

Advance directives are limited only to health care instructions and directives.

A nurse educator is teaching a group of medical nurses about Kaposis sarcoma. What would the educator identify as characteristics of endemic Kaposis sarcoma? Select all that apply.

Affects people predominantly in the eastern half of Africa Affects men more than women Can progress to lymphadenopathic forms

An occupational health nurse overhears an employee talking to his manager about a 65-year-old coworker. What phenomenon would the nurse identify when hearing the employee state, He should just retire and make way for some new blood.?

Ageism

An elderly patient is brought to the emergency department with a fractured tibia. The patient appears malnourished, and the nurse is concerned about the patients healing process related to insufficient protein levels. What laboratory finding would the floor nurse prioritize when assessing for protein deficiency?

Albumin

The nurse is describing the role of plasminogen in the clotting cascade. Where in the body is plasminogen present?

All body fluids

The nurse in a pediatric ICU is caring for a child who is dying of sickle cell anemia. The childs mother has been unable to eat or sleep and can talk only about her impending loss and the guilt she feels about the childs pain and suffering. What intervention has the highest priority?

Allowing the patient to express her feelings without judging her

An oncology nurse is contributing to the care of a patient who has failed to respond appreciably to conventional cancer treatments. As a result, the care team is considering the possible use of biologic response modifiers (BRFs). The nurse should know that these achieve a therapeutic effect by what means?

Altering the immunologic relationship between the tumor and the patient

The outer layer of the epidermis provides the most effective barrier to penetration of the skin by environmental factors. Which of the following is an example of penetration by an environmental factor?

An insect bite

A nurse will conduct an influenza vaccination campaign at an extended care facility. The nurse will be administering intramuscular (IM) doses of the vaccine. Of what age-related change should the nurse be aware when planning the appropriate administration of this drug?

An older patient has less subcutaneous tissue and less muscle mass than a younger patient.

An office worker takes a cupcake that contains peanut butter. He begins wheezing, with an inspiratory stridor and air hunger and the occupational health nurse is called to the office. The nurse should recognize that the worker is likely suffering from which type of hypersensitivity?

Anaphylactic (type 1)

The nurse is providing care for a patient who has experienced a type I hypersensitivity reaction. What condition is an example of such a reaction?

Anaphylactic reaction after a bee sting

Which response is a humoral immune response?

Anaphylaxis

A junior nursing student is having an observation day in the operating room. Early in the day, the student tells the OR nurse that her eyes are swelling and she is having trouble breathing. What should the nurse suspect?

Anaphylaxis due to a latex allergy

A nurse is educating a patient about the role of B lymphocytes. The nurses description will include which of the following physiologic processes?

Antibody production

A patient has just undergone surgery for malignant melanoma. Which of the following nursing actions should be prioritized?

Anticipate the need for, and administer, appropriate analgesic medications.

You are caring for an 82-year-old man who was recently admitted to the geriatric medical unit in which you work. Since admission, he has spoken frequently of becoming a burden to his children and staying afloat financially. When planning this patients care, you should recognize his heightened risk of what nursing diagnosis?

Anxiety

A nurse is caring for a patient who has a diagnosis of bullous pemphigoid and who is being treated on the medical unit. When providing hygiene for this patient, the nurse should perform which of the following actions?

Apply cornstarch to the patients skin after bathing to facilitate mobility.

A female patient has been achieving significant improvements in her ADLs since beginning rehabilitation from the effects of a brain hemorrhage. The nurse must observe and assess the patients ability to perform ADLs to determine the patients level of independence in self-care and her need for nursing intervention. Which of the following additional considerations should the nurse prioritize?

Appraising the familys involvement in the patients ADLs.

You are admitting a patient into your rehabilitation unit after an industrial accident. The patients nursing diagnoses include disturbed sensory perception and you assess that he has decreased strength and dexterity. You know that this patient may need what to accomplish self-care?

Appropriate assistive devices

A patient is suspected of having rheumatoid arthritis and her diagnostic regimen includes aspiration of synovial fluid from the knee for a definitive diagnosis. The nurse knows that which of the following procedures will be involved?

Arthocentesis

A gerontologic nurse has been working hard to change the perceptions of the elderly, many of which are negative, by other segments of the population. What negative perceptions of older people have been identified in the literature? Select all that apply.

As not contributing to society As draining economic resources As competing with children for resources

An 80-year-old patient is brought to the clinic by her son. The son asks the nurse why his mother has gotten so many spots on her skin. What would be an appropriate response by the nurse?

As people age, they normally develop uneven pigmentation in their skin.

A patient receiving antiretroviral therapy is complaining of "not urinating enough". What is the nurses best action?

Assess BUN and creatinine

A nurse practitioner working in a dermatology clinic finds an open lesion on a patient who is being assessed. What should the nurse do next?

Assess the characteristics of the lesion.

A client with AML has pale mucous membranes and bruises on the legs. What is the primary nursing interventions?

Assess the client's hemoglobin and platelets

You are the nurse providing care for a patient who has limited mobility after a stroke. What would you do to assess the patient for contractures?

Assess the patients range of motion.

A patients low hemoglobin level has necessitated transfusion of PRBCs. Prior to administration, what action should the nurse perform?

Assess the patients vital signs to establish baselines.

An oncology patient has just returned from the postanesthesia care unit after an open hemicolectomy. This patients plan of nursing care should prioritize which of the following?

Assess the patients wound for dehiscence every 4 hours.

A nurse is providing care for a patient who has a recent diagnosis of giant cell arteritis (GCA). What aspect of physical assessment should the nurse prioritize?

Assessment for headaches and jaw pain

Your patient is receiving postoperative morphine through a patient-controlled analgesic (PCA) pump and the patients orders specify an initial bolus dose. What is your priority assessment?

Assessment for respiratory depression

A client become anxious after an intradermal injection site begins to swell. Which technique may be used to decrease anxiety in this client?

Assure the client that this is a normal reaction

An adolescent patients history of skin hyperreactivity and inflammation has been attributed to atopic dermatitis. The nurse should recognize that this patient consequently faces an increased risk of what health problem?

Asthma

As a staff member in a local hospice, a nurse deals with death and dying on a frequent basis. Where would be the safe venue for the nurse to express her feelings of frustration and grief about a patient who has recently died?

At a staff meeting

Which assessment should be completed if immune dysfunction is suspected in the neurosensory system?

Ataxia

A pediatric nurse is emotionally distraught by the death of a 9-year-old girl who received care on the unit over the course of many admissions spanning several years. What action is the most appropriate response to the nurses own grief?

Attend the patients memorial service.

An oncology patient has begun to experience skin reactions to radiation therapy, prompting the nurse to make the diagnosis Impaired Skin Integrity: erythematous reaction to radiation therapy. What intervention best addresses this nursing diagnosis?

Avoid rubbing or scratching the affected area

An older adult resident of a long-term care facility has been experiencing generalized pruritus that has become more severe in recent weeks. What intervention should the nurse add to this residents plan of care?

Avoid using hot water during the patients baths.

An oncology nurse is caring for a patient who has developed erythema following radiation therapy. What should the nurse instruct the patient to do?

Avoid using soap on the treatment area.

A nurse is providing discharge education to a patient who has recently been diagnosed with a bleeding disorder. What topic should the nurse prioritize when teaching this patient?

Avoiding activities that carry a risk for injury

A patients most recent laboratory results show a slight decrease in potassium. The physician has opted to forego drug therapy but has suggested increasing the patients dietary intake of potassium. Which of the following would be a good source of potassium?

Bananas

You are assessing an 86-year-old postoperative patient who has an unexpressive, stoic demeanor. When you enter the room, the patient is curled into the fetal position and your assessment reveals that his vital signs are elevated and he is diaphoretic. You ask the patient what his pain level is on a 0-to-10 scale that you explained to the patient prior to surgery. The patient indicates a pain level of three or so. You review your pain-management orders and find that all medications are ordered PRN. How would you treat this patients pain?

Believe what the patient says, reinforce education, and reassess often.

A nurse practitioner is seeing a 16-year-old male patient who has come to the dermatology clinic for treatment of acne. The nurse practitioner would know that the treatment may consist of which of the following medications?

Benzoyl peroxide and erythromycin (Benzamycin)

Diagnosis of Kaposi's sarcoma (KS) is made by

Biopsy

Which condition is associated with impaired immunity relating to the aging client?

Breakdown and thinning of the skin

The nurse is performing an initial assessment of an older adult resident who has just relocated to the long-term care facility. During the nurses interview with the patient, she admits that she drinks around 20 ounces of vodka every evening. What types of cancer does this put her at risk for? Select all that apply.

Breast cancer Esophageal cancer Liver cancer

Agammaglobulinemia

Bruton's disease

Agammaglobulinemia is also known as

Bruton's disease

The most important consideration in decisions to initiate antiretroviral therapy is

CD4 counts

An unresponsive Caucasian patient has been brought to the emergency room by EMS. While assessing this patient, the nurse notes that the patients face is a cherry-red color. What should the nurse suspect?

Carbon monoxide poisoning

Patient with paroxysmal hemoglobinuria complaining of headache and weakness of the right arm and leg. Which health complication should the nurse assess with this patient?

Cerebral venous thrombosis

You have just received report on a 27-year-old woman who is coming to your unit from the emergency department with a torn meniscus. You review her PRN medications and see that she has an NSAID (ibuprofen) ordered every 6 hours. If you wanted to implement preventive pain measures when the patient arrives to your unit, what would you do?

Check for allergies, use a pain scale to assess the patients pain, and offer the ibuprofen every 6 hours until the patient is discharged.

A nurse is aware of the need to assess patients risks for anaphylaxis. What health care procedure constitutes the highest risk for anaphylaxis?

Computed tomography with contrast solution

A home care nurse performs the initial visit to a patient who is soon being discharged from a rehabilitation facility. This initial visit is to assess what the patient can do and to see what he will need when discharged home. What does this help ensure for the patient?

Continuity of care

A patient is receiving a transfusion of packed red blood cells. Shortly after initiation of the transfusion, the patient begins to exhibit signs and symptoms of a transfusion reaction. The patient is suffering from which type of hypersensitivity?

Cytotoxic (type II)

A nurse is explaining the importance of sunlight on the skin to a woman with decreased mobility who rarely leaves her house. The nurse would emphasize that ultraviolet light helps to synthesize what vitamin?

D

The baroreceptors, located in the left atrium and in the carotid and aortic arches, respond to changes in the circulating blood volume and regulate sympathetic and parasympathetic neural activity as well as endocrine activities. Sympathetic stimulation constricts renal arterioles, causing what effect?

Decrease in glomerular filtration

You are the case manager for a 35-year-old man being seen at a primary care clinic for chronic low back pain. When you meet with the patient, he says that he is having problems at work; in the past year he has been absent from work about once every 2 weeks, is short-tempered with other workers, feels tired all the time, and is worried about losing his job. You are developing this patients plan of care. On what should the goals for the plan of care focus?

Decrease the time lost from work to increase the quality of interpersonal relationships and decrease anxiety.

A 93-year-old male patient with failure to thrive has begun exhibiting urinary incontinence. When choosing appropriate interventions, you know that various age-related factors can alter urinary elimination patterns in elderly patients. What is an example of these factors?

Decreased muscle tone

Allopurinol (Zyloprim) has been ordered for a patient receiving treatment for gout. The nurse caring for this patient knows to assess the patient for bone marrow suppression, which may be manifested by which of the following diagnostic findings?

Decreased platelets

A patient has recently been diagnosed with advanced malignant melanoma and is scheduled for a wide excision of the tumor on her chest. In writing the plan of care for this patient, what major nursing diagnosis should the nurse include?

Deficient Knowledge about Early Signs of Melanoma

In the past three to four decades, nursing has moved into the forefront in providing care for the dying. Which phenomenon has most contributed to this increased focus of care of the dying?

Demographic changes in the population

A nurse is assessing the skin of a patient who has been diagnosed with bacterial cellulitis on the dorsal portion of the great toe. When reviewing the patients health history, the nurse should identify what comorbidity as increasing the patients vulnerability to skin infections?

Diabetes

A nurse is assessing a teenage patient with acne vulgaris. The patients mother states, I keep telling him that this is what happens when you eat as much chocolate as he does. What aspect of the pathophysiology of acne should inform the nurses response?

Diet is thought to play a minimal role in the development of acne.

A gerontologic nurse is basing the therapeutic programs at a long-term care facility on Millers Functional Consequences Theory. To actualize this theory of aging, the nurse should prioritize what task?

Differentiating between age-related changes and modifiable risk factors

The nurse is orienting a new nurse to the oncology unit. When reviewing the safe administration of antineoplastic agents, what action should the nurse emphasize?

Dispose of the antineoplastic wastes in the hazardous waste receptacle.

A patients health assessment has resulted in a diagnosis of alopecia areata. What nursing diagnosis should the nurse most likely associate with this health problem?

Disturbed Body Image

A patient has a diagnosis of seborrhea and has been referred to the dermatology clinic, where the nurse contributes to care. When planning this patients care, the nurse should include which of the following nursing diagnoses?

Disturbed Body Image Related to Excess Sebum Production

A client with multiple myeloma is complaining about pain. What instructions will the nurse give the client to help to reduce pain during activity?

Do not lift more than 10 pounds

A nurse provides care on a bone marrow transplant unit and is preparing a female patient for a hematopoietic stem cell transplantation (HSCT) the following day. What information should the nurse emphasize to the patients family and friends?

Do not visit if youve had a recent infection.

You are caring for a patient who has just been told that his illness is progressing and nothing more can be done for him. After the physician leaves, the patient asks you to stay with him for a while. The patient becomes tearful and tries several times to say something, but cannot get the words out. What would be an appropriate response for you to make at this time?

Do you need more time to think about this?

A patient is diagnosed with atrial fibrillation and the physician orders Coumadin (warfarin). For what skin lesion should the nurse monitor this patient?

Ecchymosis

A nurse is caring for a client undergoing evaluation for possible immune system disorders. Which intervention will best help support the client throughout the diagnostic process?

Educate the client about the diagnostic procedures and answer their questions about the possible diagnosis

A public health nurse is participating in a health promotion campaign that has the goal of improving outcomes related to skin cancer in the community. What action has the greatest potential to achieve this goal?

Educating participants about the early signs and symptoms of skin cancer

The case manager is working with an 84-year-old patient newly admitted to a rehabilitation facility. When developing a care plan for this older adult, which factors should the nurse identify as positive attributes that benefit coping in this age group? Select all that apply.

Effective adaptation skills Increased life experience Resiliency during change

A gerontologic nurse is overseeing the care that is provided in a large, long-term care facility. The nurse is educating staff about the significant threat posed by influenza in older, frail adults. What action should the nurse prioritize to reduce the incidence and prevalence of influenza in the facility?

Ensure that residents receive influenza vaccinations in the fall of each year.

When caring for a patient with toxic epidermal necrolysis (TEN), the critical care nurse assesses frequently for high fever, tachycardia, and extreme weakness and fatigue. The nurse is aware that these findings are potential indicators of what? Select all that apply.

Epidermal necrosis Increased metabolic needs Possible gastrointestinal mucosal sloughing

A patient on the oncology unit is receiving carmustine, a chemotherapy agent, and the nurse is aware that a significant side effect of this medication is thrombocytopenia. Which symptom should the nurse assess for in patients at risk for thrombocytopenia?

Epistaxis (nose bleed)

A patient with poorly controlled diabetes has developed end-stage renal failure and consequent anemia. When reviewing this patients treatment plan, the nurse should anticipate the use of what drug?

Epoetin alfa

A client who is being treated for AML has bruises on both legs. What is the nurse's most appropriate action?

Evaluate the clients platelet count

You are the rehabilitation nurse caring for a 25-year-old patient who suffered extensive injuries in a motorcycle accident. During each patient contact, what action should you perform most frequently?

Evaluate the patients positioning.

A nurse is assessing a patient with rheumatoid arthritis. The patient expresses his intent to pursue complementary and alternative therapies. What fact should underlie the nurses response to the patient?

Evidence shows minimal benefits from most CAM therapies.

Diagnostic testing has been ordered to differentiate between normal anion gap acidosis and high anion gap acidosis in an acutely ill patient. What health problem typically precedes normal anion gap acidosis?

Excessive administration of chloride

You are the nurse caring for a patient who is to receive IV daunorubicin, a chemotherapeutic agent. You start the infusion and check the insertion site as per protocol. During your most recent check, you note that the IV has infiltrated so you stop the infusion. What is your main concern with this infiltration?

Extravasation of the medication

A patient with polymyositisis experiencing challenges with activities of daily living as a result of proximal muscle weakness. What is the most appropriate nursing action?

Facilitate referrals to occupational and physical therapy

A 47-year-old patient who has come to the physicians office for his annual physical is being assessed by the office nurse. The nurse who is performing routine health screening for this patient should be aware that one of the first physical signs of aging is what?

Failing eyesight, especially close vision

After contributing to the care of several patients who died in the hospital, the nurse has identified some lapses in the care that many of these patients received toward the end of their lives. What have research studies identified as a potential deficiency in the care of the dying in hospital settings?

Families needs for information and support often go unmet.

A patient who has been newly diagnosed with systemic lupus erythematosus (SLE) has been admitted to the medical unit. Which of the following nursing diagnoses is the most plausible inclusion in the plan of care?

Fatigue Related to Anemia

A patient has just been told by his physician that he has scleroderma. The physician tells the patient that he is going to order some tests to assess for systemic involvement. The nurse knows that priority systems to be assessed include what?

Gastrointestinal

A patient with rheumatic disease is complaining of stomatitis. The nurse caring for the patient should further assess the patient for the adverse effects of what medications?

Gold-containing compounds

A patient comes to the clinic complaining of a red rash of small, fluid-filled blisters and is suspected of having herpes zoster. What presentation is most consistent with herpes zoster?

Grouped vesicles in linear patches along a dermatome

The nurse in the medical ICU is caring for a patient who is in respiratory acidosis due to inadequate ventilation. What diagnosis could the patient have that could cause inadequate ventilation?

Guillain-Barr syndrome

A nurse in the emergency department (ED) is triaging a 5-year-old who has been brought to the ED by her parents for an outbreak of urticaria. What would be the most appropriate question to ask this patient and her family?

Has she eaten any new foods today?

You are the nurse caring for an 85-year-old patient who has been hospitalized for a fractured radius. The patients daughter has accompanied the patient to the hospital and asks you what her father can do for his very dry skin, which has become susceptible to cracking and shearing. What would be your best response?

He should likely take showers rather than baths, if possible.

A nurse has been asked to become involved in the care of an adult patient in his fifties who has experienced a new onset of urinary incontinence. During what aspect of the assessment should the nurse explore physiologic risk factors for elimination problems?

Health history

The rehabilitation nurse is working closely with a patient who has a new orthosis following a knee injury. What are the nurses responsibilities to this patient? Select all that apply.

Help the patient learn to apply and remove the orthosis. Teach the patient how to care for the skin that comes in contact with the orthosis. Assist the patient in learning how to move the affected body part correctly. Collaborate with the physical therapist to set goals for care.

Gerontologic nursing is a specialty area of nursing that provides care for the elderly in our population. What goal of care should a gerontologic nurse prioritize when working with this population?

Helping older adults use their strengths to optimize independence

A patient has received a diagnosis of irritant contact dermatitis. What action should the nurse prioritize in the patients subsequent care?

Helping the patient identify and avoid the offending agent

The nurse is working with a rehabilitation patient who has a deficit in mobility following a skiing accident. The nurse knows that preparation for ambulation is extremely important. What nursing action will best provide the foundation of preparation for ambulation?

Helping the patient perform frequent exercise

A nurse is working with a patient who has a diagnosis of Cushing syndrome. When completing a physical assessment, the nurse should specifically observe for what integumentary manifestation?

Hirsutism

As the population of the United States ages, research has shown that this aging will occur across all racial and ethnic groups. A community health nurse is planning an initiative that will focus on the group in which the aging population is expected to rise the fastest. What group should the nurse identify?

Hispanics

The nurse is assessing the patient for the presence of a Chvosteks sign. What electrolyte imbalance would a positive Chvosteks sign indicate?

Hypocalcemia

A patient with a family history of allergies has suffered an allergic response based on a genetic predisposition. This atopic response is usually mediated by what immunoglobulin?

Immunoglobulin E

A 35-year-old kidney transplant patient comes to the clinic exhibiting new skin lesions. The diagnosis is Kaposis sarcoma. The nurse caring for this patient recognizes that this is what type of Kaposis sarcoma?

Immunosuppression-related

An elderly patient has come in to the clinic for her twice-yearly physical. The patient tells the nurse that she is generally enjoying good health, but that she has been having occasional episodes of constipation over the past 6 months. What intervention should the nurse first suggest?

Increase daily intake of water.

A nurse is planning the care of a patient who has a long history of chronic pain, which has only recently been diagnosed as fibromyalgia. What nursing diagnosis is most likely to apply to this womans care needs?

Ineffective Role Performance Related to Pain

A patient lives with a diagnosis of sickle cell anemia and receives frequent blood transfusions. The nurse should recognize the patients consequent risk of what complication of treatment?

Iron overload

A nurse is planning discharge teaching for an 80-year-old patient with mild short-term memory loss. The discharge teaching will include how to perform basic wound care for the venous ulcer on his lower leg. When planning the necessary health education for this patient, what should the nurse plan to do?

Keep teaching periods short.

A patient comes to the dermatology clinic requesting the removal of a port-wine stain on his right cheek. The nurse knows that the procedure especially useful in treating cutaneous vascular lesions such as port-wine stains is what?

Laser treatment

The presence of a gerontologic advanced practice nurse in a long-term care facility has proved beneficial to both the patients and the larger community in which they live. Nurses in this advanced practice role have been shown to cause what outcome?

Less deterioration takes place in the overall health of patients.

Through the process of hematopoiesis, stem cells differentiate into either myeloid or lymphoid stem cells. Into what do myeloid stem cells further differentiate? Select all that apply.

Leukocytes Platelets Erythrocytes

A nurse is educating a patient with gout about lifestyle modifications that can help control the signs and symptoms of the disease. What recommendation should the nurse make?

Limiting intake of alcohol

The nurse is caring for a patient who has developed scar tissue in many of the areas that normally produce blood cells. What organs can become active in blood cell production by the process of extramedullary hematopoiesis?

Liver and spleen

A patients most recent diagnostic imaging has revealed that his lung cancer has metastasized to his bones and liver. What is the most likely mechanism by which the patients cancer cells spread?

Lymphatic circulation

Which is an action of cytotoxic T cells?

Lyse cells infected with virus

While waiting to see the physician, a patient shows the nurse skin areas that are flat, nonpalpable, and have had a change of color. The nurse recognizes that the patient is demonstrating what?

Macules

You are the nurse creating the care plan for a patient newly admitted to your rehabilitation unit. The patient is an 82-year-old patient who has had a stroke but who lived independently until this event. What is a goal that you should include in this patients nursing care plan?

Maintain joint mobility.

A client has completed induction therapy and has diarrhea and severe mucositis. What is the appropriate nursing goal?

Maintain nutrition

A patient is being transferred from a rehabilitation setting to a long-term care facility. During this process, the nurse has utilized the referral system? Using this system achieves what goal of the patients care?

Maintaining continuity of the patients care

An oncology nurse educator is providing health education to a patient who has been diagnosed with skin cancer. The patients wife has asked about the differences between normal cells and cancer cells. What characteristic of a cancer cell should the educator cite?

Malignant cells contain proteins called tumor-specific antigens.

While performing an initial assessment of a patient admitted with appendicitis, the nurse observes an elevated blue-black lesion on the patients ear. The nurse knows that this lesion is consistent with what type of skin cancer?

Malignant melanoma

A nurse is providing care for a patient who has just been diagnosed as being in the early stage of rheumatoid arthritis. The nurse should anticipate the administration of which of the following?

Methotrexate (Rheumatrex)

A patients absolute neutrophil count (ANC) is 440/mm3. But the nurses assessment reveals no apparent signs or symptoms of infection. What action should the nurse prioritize when providing care for this patient?

Meticulous hand hygiene

You are doing discharge teaching with a patient who has hypophosphatemia during his time in hospital. The patient has a diet ordered that is high in phosphate. What foods would you teach this patient to include in his diet? Select all that apply.

Milk Poultry Poultry

The ICU nurse is caring for a patient who experienced trauma in a workplace accident. The patient is complaining of having trouble breathing with abdominal pain. An ABG reveals the following results: pH 7.28, PaCO2 50 mm Hg, HCO3 23 mEq/L. The nurse should recognize the likelihood of what acidbase disorder?

Mixed acidbase disorder

A patient is suspected of developing an allergy to an environmental substance and has been given a patch test. During the test, the patient develops fine blisters, papules, and severe itching. The nurse knows that this is indicative of what strength reaction?

Moderately positive

A nurse is caring for a patient who has allergic rhinitis. What intervention would be most likely to help the patient meet the goal of improved breathing pattern?

Modify the environment to reduce the severity of allergic symptoms.

A patient is learning about his new diagnosis of asthma with the asthma nurse. What medication has the ability to prevent the onset of acute asthma exacerbations?

Montelukast (Singulair)

The wife of a patient you are caring for asks to speak with you. She tells you that she is concerned because her husband is requiring increasingly high doses of analgesia. She states, He was in pain long before he got cancer because he broke his back about 20 years ago. For that problem, though, his pain medicine wasnt just raised and raised. What would be the nurses best response?

Much cancer pain is caused by tumor involvement and needs to be treated in a way that brings the patient relief.

A nurse is planning the care of a patient who has a diagnosis of hemophilia A. When addressing the nursing diagnosis of Acute Pain Related to Joint Hemorrhage, what principle should guide the nurses choice of interventions?

NSAIDs are contraindicated due to the risk for bleeding.

A patients intractable neuropathic pain is being treated on an inpatient basis using a multimodal approach to analgesia. After administering a recently increased dose of IV morphine to the patient, the nurse has returned to assess the patient and finds the patient unresponsive to verbal and physical stimulation with a respiratory rate of five breaths per minute. The nurse has called a code blue and should anticipate the administration of what drug?

Naloxone

A 25-year-old receives a knife wound to the leg in a hunting accident. Which type of immunity was compromised?

Natural immunity

Which type of cell is capable of directly killing invading organisms and producing cytokines?

Natural killer cells

Which type of immunity becomes active as a result of infection by specific microorganism?

Naturally acquired active immunity

A patient newly diagnosed with cancer is scheduled to begin chemotherapy treatment and the nurse is providing anticipatory guidance about potential adverse effects. When addressing the most common adverse effect, what should the nurse describe?

Nausea and vomiting

A patient has questioned the nurses administration of IV normal saline, asking whether sterile water would be a more appropriate choice than saltwater. Under what circumstances would the nurse administer electrolyte-free water intravenously?

Never, because it rapidly enters red blood cells, causing them to rupture.

A school nurse has sent home four children who show evidence of pediculosis capitis. What is an important instruction the nurse should include in the note being sent home to parents?

Nits may have to be manually removed from the childs hair shafts.

A nurse has made a referral to a grief support group, knowing that many individuals find these both comforting and beneficial after the death of a loved one. What is the most important accomplishment available by attending a grief support group?

Normalization of feelings and experiences

A nurse in the neurologic ICU has orders to infuse a hypertonic solution into a patient with increased intracranial pressure. This solution will increase the number of dissolved particles in the patients blood, creating pressure for fluids in the tissues to shift into the capillaries and increase the blood volume. This process is best described as which of the following?

Osmosis and osmolality

A new patient has come to the dermatology clinic to be assessed for a reddened rash on his abdomen. What diagnostic test would most likely be ordered to identify the causative allergen?

Patch testing

An interdisciplinary team has been working collaboratively to improve the health outcomes of a young adult who suffered a spinal cord injury in a workplace accident. Which member of the rehabilitation team is the one who determines the final outcome of the process?

Patient

A medical nurse is appraising the effectiveness of a patients current pain control regimen. The nurse is aware that if an intervention is deemed ineffective, goals need to be reassessed and other measures need to be considered. What is the role of the nurse in obtaining additional pain relief for the patient?

Patient advocate

A nurse is aware that the outer layer of the skin consists of dead cells that contain large amounts of keratin. The physiologic functions of keratin include which of the following? Select all that apply.

Physically repelling pathogens Preventing fluid loss

A nurse has cited a research study that highlights the clinical effectiveness of using placebos in the management of postsurgical patients pain. What principle should guide the nurses use of placebos in pain management?

Placebos are never recommended in the treatment of pain.

A patient has been diagnosed with a lymphoid stem cell defect. This patient has the potential for a problem involving which of the following?

Plasma cells

A patients wound has begun to heal and the blood clot which formed is no longer necessary. When a blood clot is no longer needed, the fibrinogen and fibrin will be digested by which of the following?

Plasmin

An individual has accidentally cut his hand, immediately initiating the process of hemostasis. Following vasoconstriction, what event in the process of hemostasis will take place?

Platelets will aggregate at the injury site.

A patient on the medical unit is dying and the nurse has determined that the familys psychosocial needs during the dying process need to be addressed. What is a cause of many patient care dilemmas at the end of life?

Poor communication between the family and the care team

You are the nurse caring for an elderly adult who is bedridden. What intervention would you include in the care plan that would most effectively prevent pressure ulcers?

Post a turning schedule at the patients bedside and ensure staff adherence.

A patient undergoing a hip replacement has autologous blood on standby if a transfusion is needed. What is the primary advantage of autologous transfusions?

Prevention of viral infections from another persons blood

The nurse is caring for a 39-year-old woman with a family history of breast cancer. She requested a breast tumor marking test and the results have come back positive. As a result, the patient is requesting a bilateral mastectomy. This surgery is an example of what type of oncologic surgery?

Prophylactic surgery

A hospice nurse is caring for a 22-year-old with a terminal diagnosis of leukemia. When updating this patients plan of nursing care, what should the nurse prioritize?

Providing realistic emotional preparation for death

A new patient presents at the clinic and the nurse performs a comprehensive health assessment. The nurse notes that the patients fingernail surfaces are pitted. The nurse should suspect the presence of what health problem?

Psoriasis

A nurse in a dermatology clinic is reading the electronic health record of a new patient. The nurse notes that the patient has a history of a primary skin lesion. What is an example of a primary skin lesion?

Pustule

The nurse is describing some of the major characteristics of cancer to a patient who has recently received a diagnosis of malignant melanoma. When differentiating between benign and malignant cancer cells, the nurse should explain differences in which of the following aspects? Select all that apply.

Rate of growth Ability to cause death Ability to spread

The nurse is preparing to care for a patient who has scleroderma. The nurse refers to resources that describe CREST syndrome. Which of the following is a component of CREST syndrome?

Raynauds phenomenon

A home care nurse is making an initial visit to a 68-year-old man. The nurse finds the man tearful and emotionally withdrawn. Even though the man lives alone and has no family, he has been managing well at home until now. What would be the most appropriate action for the nurse to take?

Reassess the patients psychosocial status and make the necessary referrals

A patient is admitted to the intensive care unit with what is thought to be toxic epidermal necrolysis (TEN). When assessing the health history of the patient, the nurse would be alert to what precipitating factor?

Recent administration of new medications

The nurse is caring for a patient diagnosed with cancer of the liver who has chosen to remain in his home as long as he is able. The nurse reviews the care plan for the patient and notes that it focuses on palliative measures. The nurse also notes that over the last 3 weeks, the patients condition has continued to deteriorate. What is the nurses best response to this clinical information?

Recognize that the patient is in the downward phase of chronic illness and should be reassessed.

The rehabilitation team has reaffirmed the need to maximize the independence of a patient in rehabilitation. When working toward this goal, what action should the nurse prioritize?

Recognize the importance of informal caregivers.

A patient who undergoes hemodialysis three times weekly is on a fluid restriction of 1000 mL/day. The nurse sees the patient drinking a 355-mL (12 ounce) soft drink after the patient has already reached the maximum intake of fluid for the day. What action should the nurse take?

Reinforce the importance of the fluid restriction and document the teaching and the intake of extra fluid

A nurse is caring for a patient whose chemical injury has necessitated a skin graft to his left hand. The nurse enters the room and observes that the patient is performing active range of motion (ROM) exercises with the affected hand. How should the nurse best respond?

Remind the patient of the need to immobilize the graft to facilitate healing.

After the completion of testing, a childs allergies have been attributed to her familys cat. When introducing the family to the principles of avoidance therapy, the nurse should promote what action?

Removing the cat from the familys home

A medical-surgical nurse is teaching a patient about the health implications of her recently diagnosed type 2 diabetes. The nurse should teach the patient to be proactive with her glycemic control in order to reduce her risk of what health problem?

Renal failure

Which condition is associated with impaired immunity relating to the aging client?

Renal function decreases

A patient who is being treated for pneumonia starts complaining of sudden shortness of breath. An arterial blood gas (ABG) is drawn. The ABG has the following values: pH 7.21, PaCO2 64 mm Hg, HCO3 = 24 mm Hg. What does the ABG reflect?

Respiratory acidosis

A patient with a longstanding diagnosis of generalized anxiety disorder presents to the emergency room. The triage nurse notes upon assessment that the patient is hyperventilating. The triage nurse is aware that hyperventilation is the most common cause of which acidbase imbalance?

Respiratory alkalosis

Which adverse effect should the nurse closely monitor in a client who takes immunosuppressive drugs?

Respiratory or urinary system infections

Which adverse effects should the nurse closely monitor in a patient who has secondary immunodeficiencies due to immunosuppressive therapy?

Respiratory or urinary system infections

A patients most recent blood work reveals low levels of albumin. This assessment finding should suggest the possibility of what nursing diagnosis?

Risk for imbalanced fluid volume related to low albumin

The nurse on a bone marrow transplant unit is caring for a patient with cancer who is preparing for HSCT. What is a priority nursing diagnosis for this patient?

Risk for infection related to altered immunologic response

Patients who are enrolled in hospice care through Medicare are often felt to suffer unnecessarily because they do not receive adequate attention for their symptoms of the underlying illness. What factor most contributes to this phenomenon?

Rules concerning completion of all cure-focused medical treatment

An African American is admitted to the medical unit with liver disease. To correctly assess this patient for jaundice, on what body area should the nurse look for yellow discoloration?

Sclerae

A 40-year-old woman was diagnosed with Raynauds phenomenon several years earlier and has sought care because of a progressive worsening of her symptoms. The patient also states that many of her skin surfaces are stiff, like the skin is being stretched from all directions. The nurse should recognize the need for medical referral for the assessment of what health problem?

Scleroderma

A public health nurse has formed an interdisciplinary team that is developing an educational program entitled Cancer: The Risks and What You Can Do About Them. Participants will receive information, but the major focus will be screening for relevant cancers. This program is an example of what type of health promotion activity?

Secondary prevention

The home health nurse is developing a plan of care for a patient who will be managing his chronic pain at home. Using the nursing process, on which concepts should the nurse focus the patient teaching?

Self-care and safety

A man suffers a leg wound which causes minor blood loss. As a result of bleeding, the process of primary hemostasis is activated. What occurs in primary hemostasis?

Severed blood vessels constrict.

A community health nurse has drafted a program that will address the health promotion needs of members of the community who live with one or more disabilities. Which of the following areas of health promotion education is known to be neglected among adults with disabilities?

Sexual health

You are caring for a patient with a secondary diagnosis of hypermagnesemia. What assessment finding would be most consistent with this diagnosis?

Shallow respirations

You are caring for a patient with a history of chronic angina. The patient tells you that after breakfast he usually takes a shower and shaves. It is at this time, the patient says, that he tends to experience chest pain. What might you counsel the patient to do to decrease the likelihood of angina in the morning?

Shower in the evening and shave before breakfast.

Fresh-frozen plasma (FFP) has been ordered for a hospital patient. Prior to administration of this blood product, the nurse should prioritize what patient education?

Signs and symptoms of a transfusion reaction

The nurses review of a patients most recent blood work reveals a significant increase in the number of band cells. The nurses subsequent assessment should focus on which of the following?

Signs and symptoms of infection

A patient with a suspected malignant melanoma is referred to the dermatology clinic. The nurse knows to facilitate what diagnostic test to rule out a skin malignancy?

Skin biopsy

The public health nurse is presenting a health-promotion class to a group at a local community center. Which intervention most directly addresses the leading cause of cancer deaths in North America?

Smoking cessation

An older adult patient is diagnosed with a vitamin D deficiency. What would be an appropriate recommendation by the nurse?

Spend time outdoors at least twice per week

A nurse has included the nursing diagnosis of Risk for Latex Allergy Response in a patients plan of care. The presence of what chronic health problem would most likely prompt this diagnosis?

Spina bifida

A critical care nurse is caring for a patient with autoimmune hemolytic anemia. The patient is not responding to conservative treatments, and his condition is now becoming life threatening. The nurse is aware that a treatment option in this case may include what?

Splenectomy

You have been referred to the care of an extended care resident who has been diagnosed with a stage III pressure ulcer. You are teaching staff at the facility about the role of nutrition in wound healing. What would be the best meal choice for this patient?

Steak, baked potato, spinach and strawberry salad

An older adult client is exhibiting many of the characteristic signs and symptoms of iron deficiency. In addition to a complete blood count, what diagnostic assessment should the nurse anticipate?

Stool for occult blood

The nurse caring for a patient receiving a transfusion notes that 15 minutes after the infusion of packed red blood cells (PRBCs) has begun, the patient is having difficulty breathing and complains of severe chest tightness. What is the most appropriate initial action for the nurse to take?

Stop the transfusion immediately.

While a patient is receiving IV doxorubicin hydrochloride for the treatment of cancer, the nurse observes swelling and pain at the IV site. The nurse should prioritize what action?

Stopping the administration of the drug immediately

A patient with an exceptionally low body mass index has been admitted to the emergency department with signs and symptoms of hypothermia. The nurse should know that this patients susceptibility to heat loss is related to atrophy of what skin component?

Subcutaneous tissue

The nurse is caring for a patient with an advanced stage of breast cancer and the patient has recently learned that her cancer has metastasized. The nurse enters the room and finds the patient struggling to breath and the nurses rapid assessment reveals that the patients jugular veins are distended. The nurse should suspect the development of what oncologic emergency?

Superior vena cava syndrome (SVCS)

The nurse is admitting a 52-year-old father of four into hospice care. The patient has a diagnosis of Parkinsons disease, which is progressing rapidly. The patient has made clear his preference to receive care at home. What interventions should the nurse prioritize in the plan of care?

Supporting the patients and familys values and choices

A patient with squamous cell carcinoma has been scheduled for treatment of this malignancy. The nurse should anticipate that treatment for this type of cancer will primarily consist of what intervention?

Surgical excision

A nurse is providing care for a patient who has a rheumatic disorder. The nurses comprehensive assessment includes the patients mood, behavior, LOC, and neurologic status. What is this patients most likely diagnosis?

Systemic lupus erythematosus (SLE)

A patient with a diagnosis of gastric cancer has been unable to tolerate oral food and fluid intake and her tumor location precludes the use of enteral feeding. What intervention should the nurse identify as best meeting this patients nutritional needs?

TPN administered via a peripherally inserted central catheter

A patient with SLE asks the nurse why she has to come to the office so often for check-ups. What would be the nurses best response?

Taking care of you in the best way involves monitoring your disease activity and how well the prescribed treatment is working.

A 39-year-old patient with paraplegia has been admitted to the hospital for the treatment of a sacral ulcer. The nurse is aware that the patient normally lives alone in an apartment and manages his ADLs independently. Before creating the patients plan of care, how should the nurse best identify the level of assistance that the patient will require in the hospital?

Talk with the patient about the type and level of assistance that he desires.

A nurse is caring for a patient whose skin cancer will soon be removed by excision. Which of the following actions should the nurse perform?

Teach the patient about self-care after treatment.

A 21-year-old male has just been diagnosed with a spondyloarthropathy. What will be a priority nursing intervention for this patient?

Teaching about symptom management

A nurse is working with a family whose 5 year-old daughter has been diagnosed with impetigo. What educational intervention should the nurse include in this familys care?

Teaching about the importance of maintaining high standards of hygiene

The hospice nurse is caring for a 45-year-old mother of three young children in the patients home. During the most recent visit, the nurse has observed that the patient has a new onset of altered mental status, likely resulting from recently diagnosed brain metastases. What goal of nursing interventions should the nurse identify?

Teaching family members how to interact with, and ensure safety for, the patient with impaired cognition

A nurse is leading a health promotion workshop that is focusing on cancer prevention. What action is most likely to reduce participants risks of basal cell carcinoma (BCC)?

Teaching participants to limit their sun exposure

Traditionally, nurses have been involved with tertiary cancer prevention. However, an increasing emphasis is being placed on both primary and secondary prevention. What would be an example of primary prevention?

Teaching patients to wear sunscreen

What intervention is appropriate before the patient starts on efavirenz (EFV, Sustiva) therapy?

Testing for Steven-Johnson syndrome potential

A patient has been transferred to a rehabilitative setting from an acute care unit. What is the most important reason for the nurse to begin a program for activities of daily living (ADLs) as soon as the patient is admitted to a rehabilitation facility?

The ability to perform ADLs may be the key to reentry into the community.

A 60-year-old patient who has diabetes had a below-knee amputation 1 week ago. The patient asks why does it still feel like my leg is attached, and why does it still hurt? The nurse explains neuropathic pain in terms that are accessible to the patient. The nurse should describe what pathophysiologic process?

The abnormal reorganization of the nervous system

You are the nurse caring for an elderly patient with cardiovascular disease. The patient comes to the clinic with a suspected respiratory infection and is diagnosed with pneumonia. As the nurse, what do you know about the altered responses of older adults?

The altered responses of older adults reinforce the need for the nurse to monitor all body systems to identify possible systemic complications.

You are caring for a patient with late-stage Alzheimers disease. The patients wife tells you that the patient has now become completely dependent and that she feels guilty if she takes any time for herself. What outcomes would be appropriate for the nurse to develop to assist the patients wife?

The caregiver distinguishes essential obligations from those that can be controlled or limited.

Two units of PRBCs have been ordered for a patient who has experienced a GI bleed. The patient is highly reluctant to receive a transfusion, stating, Im terrified of getting AIDS from a blood transfusion. How can the nurse best address the patients concerns?

The chances of contracting AIDS from a blood transfusion in the United States are exceedingly low.

As a member of the rehabilitation team, the nurse is conscious of the need to perform the nursing role in collaboration with the other members of the team. Which of the following variables has the greatest bearing on the nurses choice of actions and interventions during rehabilitative care?

The circumstances of the patient

The nurse is assessing several clients. Which client does the nurse determine is most likely to have Hodgkin lymphoma?

The client with enlarged lymph nodes in the neck

A patient is being treated in the ICU after a medical error resulted in an acute hemolytic transfusion reaction. What was the etiology of this patients adverse reaction?

The donor blood was incompatible with that of the patient.

A patients rheumatoid arthritis (RA) has failed to respond appreciably to first-line treatments and the primary care provider has added prednisone to the patients drug regimen. What principle will guide this aspect of the patients treatment?

The drug should be used for as short a time as possible.

A nurse has asked the nurse educator if there is any way to predict the severity of a patients anaphylactic reaction. What would be the nurses best response?

The faster the onset of symptoms, the more severe the reaction

A nurse at an allergy clinic is providing education for a patient starting immunotherapy for the treatment of allergies. What education should the nurse prioritize?

The importance of keeping appointments for desensitization procedures

Which of the following would be inaccurate information pertaining to SCID?

The inheritance of SCID can be autosomal dominant

A medical nurse educator is reviewing a patients recent episode of metabolic acidosis with members of the nursing staff. What should the educator describe about the role of the kidneys in metabolic acidosis?

The kidneys excrete hydrogen ions and conserve bicarbonate ions to help restore balance.

A nurse is planning care for a nephrology patient with a new nursing graduate. The nurse states, A patient in renal failure partially loses the ability to regulate changes in pH. What is the cause of this partial inability?

The kidneys regenerate and reabsorb bicarbonate to maintain a stable pH.

A 30-year-old male patient has just returned from the operating room after having a flap done following a motorcycle accident. The patients wife asks the nurse about the major complications following this type of surgery. What would be the nurses best response?

The major complication is when the blood supply fails and the tissue in the flap dies.

A 73-year-old man comes into the emergency department (ED) by ambulance after slipping on a small carpet in his home. The patient fell on his hip with a resultant fracture. He is alert and oriented; his pupils are equal and reactive to light and accommodation. His heart rate is elevated, he is anxious and thirsty, a Foley catheter is placed, and 40 mL of urine is present. What is the nurses most likely explanation for the low urine output?

The man is having a sympathetic reaction, which has stimulated the reninangiotensinaldosterone system that results in diminished urine output.

A patients low prothrombin time (PT) was attributed to a vitamin K deficiency and the patients PT normalized after administration of vitamin K. When performing discharge education in an effort to prevent recurrence, what should the nurse emphasize?

The need for adequate nutrition

A child has been diagnosed with a severe walnut allergy after suffering an anaphylactic reaction. What is a priority for health education?

The need for the parents to carry an epinephrine pen

A patient is scheduled for a splenectomy. During discharge education, what teaching point should the nurse prioritize?

The need to report any signs or symptoms of infection promptly

When planning the skin care of a patient with decreased mobility, the nurse is aware of the varying thickness of the epidermis. At what location is the epidermal layer thickest?

The palms of the hands

An adult oncology patient has a diagnosis of bladder cancer with metastasis and the patient has asked the nurse about the possibility of hospice care. Which principle is central to a hospice setting?

The patient and family should be viewed as a single unit of care.

The nurse is caring for an older adult patient who is receiving rehabilitation following an ischemic stroke. A review of the patients electronic health record reveals that the patient usually defers her self-care to family members or members of the care team. What should the nurse include as an initial goal when planning this patients subsequent care?

The patient will demonstrate independent self-care.

You are caring for a 35-year-old man whose severe workplace injuries necessitate bilateral below-the-knee amputations. How can you anticipate that the patient will respond to this news?

The patient will experience grief in an individualized manner.

A nurse is caring for a 78-year-old patient with a history of osteoarthritis (OA). When planning the patients care, what goal should the nurse include?

The patient will express satisfaction with her ability to perform ADLs.

You are the nurse caring for the 25-year-old victim of a motor vehicle accident with a fractured pelvis and a ruptured bladder. The nurses aide (NA) tells you that she is concerned because the patients resting heart rate is 110 beats per minute, her respirations are 24 breaths per minute, temperature is 99.1F axillary, and the blood pressure is 125/85 mm Hg. What other information is most important as you assess this patients physiologic status?

The patients rating of her pain

A patient newly diagnosed with thrombocytopenia is admitted to the medical unit. After the admission assessment, the patient asks the nurse to explain the disease. What should the nurse explain to this patient?

There could be decreased production of platelets.

The home health nurse is making an initial home visit to a 76-year-old widower. The patient takes multiple medications for the treatment of varied chronic health problems. The patient states that he has also begun taking some herbal remedies. What should the nurse be sure to include in the patients teaching?

There is a need to inform his physician and pharmacist about the herbal remedies.

The mother of a cancer patient comes to the nurse concerned with her daughters safety. She states that her daughters morphine dose that she needs to control her pain is getting higher and higher. As a result, the mother is afraid that her daughter will overdose. The nurse educates the mother about what aspect of her pain management?

There is no absolute maximum opioid dose and her daughter is becoming more tolerant to the drug.

During the care conference for a patient who has multiple chronic conditions, the case manager has alluded to the principles of the interface model of disability. What statement is most characteristic of this model?

This patient can be empowered and doesnt have to be dependent.

A major manifestation of Wiskott-Aldrich syndrome included which of the following?

Thrombocytopenia

T-Cell deficiency occurs when which gland fails to develop normally during embryogenesis?

Thymus

Which adverse effect should a client who is receiving IVIG report immediately?

Tickle in the throat

The nurse administers IV gamma-globulin infusion, which complaint by the client may indicate an adverse effect of the infusion?

Tightness in the chest

Your patient is 12-hours post ORIF right ankle. The patient is asking for a breakthrough dose of analgesia. The pain-medication orders are written as a combination of an opioid analgesic and a nonsteroidal anti-inflammatory drug (NSAID) given together. What is the primary rationale for administering pain medication in this manner?

To achieve better pain control than with one medication alone

A patient is diagnosed with giant cell arteritis (GCA) and is placed on corticosteroids. A concern for this patient is that he will stop taking the medication as soon as he starts to feel better. Why must the nurse emphasize the need for continued adherence to the prescribed medication?

To avoid complications such as blindness

An adult patients current goals of rehabilitation focus primarily on self-care. What is a priority when teaching a patient who has self-care deficits in ADLs?

To provide an optimal learning environment with minimal distractions

A patient has completed the acute treatment phase of care following a stroke and the patient will now begin rehabilitation. What should the nurse identify as the major goal of the rehabilitative process?

To restore the patients ability to function independently

A patient is experiencing severe pain after suffering an electrical burn in a workplace accident. The nurse is applying knowledge of the pathophysiology of pain when planning this patients nursing care. What is the physiologic process by which noxious stimuli, such as burns, activate nociceptors?

Transduction

A patient with a recent diagnosis of ITP has asked the nurse why the care team has not chosen to administer platelets, stating, I have low platelets, so why not give me a transfusion of exactly what Im missing? How should the nurse best respond?

Transfused platelets usually arent beneficial because theyre rapidly destroyed in the body.

One of the functions of nursing care of the terminally ill is to support the patient and his or her family as they come to terms with the diagnosis and progression of the disease process. How should nurses support patients and their families during this process? Select all that apply.

Try to appreciate and understand the illness from the patients perspective. Assist patients with performing a life review. Provide interventions that facilitate end-of-life closure.

The home health nurse is performing a home visit for an oncology patient discharged 3 days ago after completing treatment for non-Hodgkin lymphoma. The nurses assessment should include examination for the signs and symptoms of what complication?

Tumor lysis syndrome (TLS)

You are the nurse caring for a postsurgical patient who is Asian-American who speaks very little English. How should you most accurately assess this patients pain?

Use a chart with English on one side of the page and the patients native language on the other so he can rate his pain.

You are caring for an adult patient who has developed a mild oral yeast infection following chemotherapy. What actions should you encourage the patient to perform? Select all that apply.

Use a lip lubricant. Use dental floss every 24 hours. Rinse the mouth with normal saline.

A nurse is caring for a patient who has been diagnosed with psoriasis. The nurse is creating an education plan for the patient. What information should be included in this plan?

Use caution when taking nonprescription medications.

You are the nurse caring for an elderly patient who has been on a bowel training program due to the neurologic effects of a stroke. In the past several days, the patient has begun exhibiting normal bowel patterns. Once a bowel routine has been well established, you should avoid which of the following?

Use of a bedpan

You are creating a nursing care plan for a patient who is hospitalized following right total hip replacement. What nursing action should you specify to prevent inward rotation of the patients hip when the patient is in a partial lateral position?

Use of an abduction pillow between the patients legs

f emphysema. The new nurses preceptor is going over the patients past lab reports with the new nurse. The nurse takes note that the patients PaCO2 has been between 56 and 64 mm Hg for several months. The preceptor asks the new nurse why they will be cautious administering oxygen. What is the new nurses best response?

Using oxygen may result in the patient developing carbon dioxide narcosis and hypoxemia.

Which diagnostic test measures HIV RNA in the plasma?

Viral load

A nurse is planning patient education for a patient being discharged home with a diagnosis of rheumatoid arthritis. The patient has been prescribed antimalarials for treatment, so the nurse knows to teach the patient to self-monitor for what adverse effect?

Visual changes

A patient is admitted to the hospital with pernicious anemia. The nurse should prepare to administer which of the following medications?

Vitamin B12

A patient with a history of atrial fibrillation has contacted the clinic saying that she has accidentally overdosed on her prescribed warfarin (Coumadin). The nurse should recognize the possible need for what antidote?

Vitamin K

While assessing a dark-skinned patient at the clinic, the nurse notes the presence of patchy, milky white spots. The nurse knows that this finding is characteristic of what diagnosis?

Vitiligo

A nurse is providing self-care education to a patient who has been receiving treatment for acne vulgaris. What instruction should the nurse provide to the patient?

Wash your face with water and gentle soap each morning and evening.

A patient with a documented history of glucose-6-phosphate dehydrogenase deficiency has presented to the emergency department with signs and symptoms including pallor, jaundice, and malaise. Which of the nurses assessment questions relates most directly to this patients hematologic disorder?

What medications have taken recently?

A young student comes to the school nurse and shows the nurse a mosquito bite. As the nurse expects, the bite is elevated and has serous fluid contained in the dermis. How would the nurse classify this lesion?

Wheal

A man and woman are in their early eighties and have provided constant care for their 44-year-old son who has Down syndrome. When planning this familys care, the nurse should be aware that the parents most likely have what concerns around what question?

Who will care for our son once were unable?

A patient has just been told that he has malignant melanoma. The nurse caring for this patient should anticipate that the patient will undergo what treatment?

Wide excision

An elderly patient has presented to the clinic with a new diagnosis of osteoarthritis. The patients daughter is accompanying him and you have explained why the incidence of chronic diseases tends to increase with age. What rationale for this phenomenon should you describe?

With age, biologic changes reduce the efficiency of body systems.

The nurse has observed that an older adult patient with a diagnosis of end-stage renal failure seems to prefer to have his eldest son make all of his health care decisions. While the family is visiting, the patient explains to you that this is a cultural practice and very important to him. How should you respond?

Work with the team to negotiate informed consent.

A 55-year-old woman is scheduled to have a chemical face peel. The nurse is aware that the patient is likely seeking treatment for which of the following?

Wrinkles near the lips and eyes

A 58-year-old male patient has been hospitalized for a wedge resection of the left lower lung lobe after a routine chest x-ray shows carcinoma. The patient is anxious and asks if he can smoke. Which statement by the nurse would be most therapeutic?

You are anxious about the surgery. Do you see smoking as helping?

You are the nurse caring for a patient who has paraplegia following a hunting accident. You know to assess regularly for the development of pressure ulcers on this patient. What rationale would you cite for this nursing action?

You know that the risk for pressure ulcers is directly related to the duration of immobility.

A treatment option for SCID includes

a stem cell transplant

Chronic mucocutaneous candidiasis

autosomal recessive disorder; all children will be carriers but may not develop disease

What microorganism is known to cause retinitis in people with HIV/AIDS?

cytomeglovirus

amprenavir (APV, Agenerase) side effect

fat redistribution

An early manifestation of HIV encephalopathy is a

headache

Which type of phagocytic disorder occurs when WBC cannot initiate an inflammatory response to infectious organisms?

hyperimmunoglobulinenemia

Which term means a lack of one or more of the five immunoglobulins?

hypogammaglobulinemia

For a client with Hodgkin lymphoma, who is at risk for ineffective airway clearance and impaired gas exchange, the nurse places the client in high Fowler's position to

increase lung expansion

Proteins formed when cells are exposed to viral or foreign agents that are capable of activating other components of the immune system are referred to as

interferons

Treatment of HIV infection

is based on the clinical condition of the patient, CD4 cell level count, and HIV RNA (viral load)

Which term refers to a form of white blood cell involved in immune response?

lymphocyte

A patient who has common variable immunodeficiency should need more frequent screening for what complication?

malignancy

Which cell of hematopoiesis is responsible for the production of red blood cells and platelets?

myeloid stem cell

Ibuprofen affects the immune system by causing

neutropenia

Which term refers to an abnormal decrease in white blood cells, red blood cells, ad platelets?

pancytopenia

What has NOT been implicated as a factor for noncompliance with antiretroviral treatment?

past substance abuse

Which stage of the immune response occurs when the differentiated lymphocytes function in either a humoral or a cellular capacity?

response stage

patient with immunodeficiency disorder

should not eat raw fruits or vegetables, all foods should be cooked thoroughly and leftovers refrigerated immediately to prevent infection

Which term is used to refer to a primitive cell that is capable to self-replication and differentiation?

stem cell

Itching in the back of the throat after IVIG was administered, the first action should be to

stop the infusion

The immune benefits of Breastfeeding are beneficial because

the dominant antibody IgA in breast milk acts by functioning as an antigen receptor in the mucosal membranes

In which response do T lymphocytes have a role?

transplant rejection

Ataxia is the term that refers to

uncoordinated muscle movement

What test will the nurse assess to determine the patients response to antiretroviral therapy?

viral load

Balance between the amount of HIV in the body and the immune response is

viral set point

The nurse is teaching a client with acute lymphocytic leukemia (ALL) about therapy. What statement should be included in the plan of care?

"Intrathecal chemotherapy is used primarily as preventive therapy."

The nurse is interacting with a family that has been caring for a client with cancer for several months. What are the best interventions to assist in relieving caregiver stress in this family?

-Allow family members to express feelings -Educate the family about medications and side effects -Suggest support for household maintenance

Which alterations in immune function can be attributed to smoking?

-Increased WBC count -Increased risk of infection -Decreased T-Cell function -Decreased natural killer cells

What assessment finding best indicates that the client has recovered from induction therapy?

Neutrophil and platelet counts within normal limits

Other than abstinence, what is the only proven method of decreasing the risk of sexually transmission of HIV infection?

consistent and correct use of condoms

Which microorganism is known to cause retinitis in people with HIV/AIDS?

cytomegalovirus

A negative HIV test result means

the body has not produced antibodies to the AIDS virus

A client with suspected multiple myeloma is reporting back pain. What is the priority nursing action?

Send the client for a spinal x-ray study

Which statement indicates the client understands teaching about induction therapy for leukemia?

"I will be in the hospital for several weeks."

Which stage of HIV infection is indicated when the results are more than 500 CD4+ lymphocytes/mm?

CDC category A: HIV asymptomatic

a 6-year-old client is diagnosed with a viral infection of the respiratory system. Which will most likely be trying to fight the antigen?

Interferons

Which type of disorder produces recurrent and persistent infection of the soft tissue, lungs, and other organs?

Primary phagocytic disorder

Which statement best describes the function of the stem cells in the bone marrow?

The produce all blood cells

Which statement accurately reflects current stem cell research?

The stem cell is known as a precursor cell that continually replenishes the body's entire supply of both red and white cells

patient with agammaglobulinemia at the beach

can play in the sand, but not the water

After chemotherapy for AML, what interventions will best help to prevent renal complications?

-Administer allopurinol -Administer rasburicase -Increase hydration

Which assessment findings support the clients diagnosis of acute myeloid leukemia?

-weakness and fatigue -enlarged lymph nodes -bone pain -petechiae

A client presents with peripheral neuropathy and hypothesia of the feet. What is the best nursing intervention?

Assess for signs of injury

A client who is undergoing chemotherapy for AML reports pain in the low back. What is the nurse's first action?

Assess renal function

Cells that attack the antigen directly by altering the cell membrane and causing cell lysis is referred to as

Cytotoxic T cells

A client has an emergency c-section to deliver her baby who has developed an infection in utero. Which immunoglobulin will be increased in the fetus at the time of birth that will actively be fighting the infection?

IgG

Which immunoglobulin assumes a major role in bloodborne and tissue infections?

IgG

Primary immunodeficiences

develop early in life after protection from maternal antibodies decreases

a patient with SCID receiving immunosuppression therapy; priority nursing care for this patient would be?

meticulous infection control precautions

Selective Immunoglobulin A Deficiency

mild genetic immune deficiency caused by a lack of immunoglobulin A (IgA), a type of antibody that protects against infections of the lining of the mouth and digestive tract

Which adverse effect should closely be monitored after foscarnet is administered to patients with CMV?

Electrolyte imbalances

A client with leukemia has developed a cough and increased fatigue. What is the primary nursing intervention?

Evaluate the client for potential infection

which nursing intervention should be incorporated into the plan of care to manage the delayed clotting process in a client with leukemia?

Apply prolonged pressure to needle sites or other sources of external bleeding

What should the nurse encourage to replace fluid and electrolyte loss in a patient with AIDS?

Liquids


Kaugnay na mga set ng pag-aaral

Chapter 7 Ownership of Real Property

View Set

History 1302 Reading quiz 2 review

View Set

Capital Gains and Losses Part 1 (Wash Sales & Related-Party Transactions)

View Set